background image

     CHAPTER  OBJECTIVES 

 ● 

  

● 

  Become familiar with the  

e

  , hybrid, and hybrid 

p models for the BJT transistor.  

  

● 

  Learn to use the equivalent model to find the important ac parameters for an amplifier.  

  

● 

  Understand the effects of a source resistance and load resistor on the overall gain and 
characteristics of an amplifier.  

  

● 

  Become aware of the general ac characteristics of a variety of important BJT 
configurations.  

  

● 

  Begin to understand the advantages associated with the two-port systems approach to 
single- and multistage amplifiers.  

  

● 

  Develop some skill in troubleshooting ac amplifier networks.    

      5.1 

 INTRODUCTION 

 ● 

 The basic construction, appearance, and characteristics of the transistor were introduced in 
 Chapter   3   . The dc biasing of the device was then examined in detail in  Chapter   4   . We now 
begin to examine the ac response of the BJT amplifier by reviewing the  models  most fre-
quently used to represent the transistor in the sinusoidal ac domain. 

 One of our first concerns in the sinusoidal ac analysis of transistor networks is the mag-

nitude of the input signal. It will determine whether  small-signal  or  large-signal   techniques 
should be applied. There is no set dividing line between the two, but the application—and 
the magnitude of the variables of interest relative to the scales of the device characteristics—
will usually make it quite clear which method is appropriate. The small-signal technique is 
introduced in this chapter, and large-signal applications are examined in  Chapter   12   . 

 There are three models commonly used in the small-signal ac analysis of transistor 

networks: the  

e

   model, the hybrid 

p model, and the hybrid equivalent model. This chapter 

introduces all three but emphasizes the  

e

   model.  

   5.2 

 AMPLIFICATION  IN  THE  AC  DOMAIN 

 ● 

 It was demonstrated in  Chapter   3    that the transistor can be employed as an amplifying device. 
That is, the output sinusoidal signal is greater than the input sinusoidal signal, or, stated 
another way, the output ac power is greater than the input ac power. The question then arises 
as to how the ac power output can be greater than the input ac power. Conservation of energy 
dictates that over time the total power output,  

o

  , of a system cannot be greater than its power 

    5 

 BJT AC Analysis 

253


background image

input,  

i

  , and that the efficiency defined by    

h = P

o

>P

i

    cannot be greater than 1. The factor 

missing from the discussion above that permits an ac power output greater than the input ac 
power is the applied dc power. It is the principal contributor to the total output power even 
though part of it is dissipated by the device and resistive elements. In other words, there is an 
“exchange” of dc power to the ac domain that permits establishing a higher output ac power. 
In fact, a  conversion efficiency  is defined by    

h = P

o(ac)

>P

i(dc)

,     where     P

o(ac)

    is the ac power 

to the load and    P

i(dc)

    is the dc power supplied. 

 Perhaps the role of the dc supply can best be described by first considering the simple 

dc network of  Fig.   5.1   . The resulting direction of flow is indicated in the figure with a plot 
of the current  i  versus time. Let us now insert a control mechanism such as that shown in 
 Fig.   5.2   . The control mechanism is such that the application of a relatively small signal to 
the control mechanism can result in a substantial oscillation in the output circuit. 

  

BJT AC ANALYSIS

254

 

I

dc

I

dc

I

dc

I

dc

I

dc

E

R

i

t

0

+

 FIG. 5.1 

          Steady  current  established  by  a 

dc  supply.    

i

T

i

c

i

T

i

T

i

T

R

i

I

dc 

i

ac

t

0

Control
mechanism

E

+

 FIG. 5.2 

         Effect of a control element on the steady-state flow of the electrical 

system  of   Fig.    5.1   .    

 That is, for this example, 

 

   i

ac(p

@p)

i

c(p

@p)

   

 and amplification in the ac domain has been established. The peak-to-peak value of the 
output current far exceeds that of the control current. 

 For the system of  Fig.   5.2   , the peak value of the oscillation in the output circuit is con-

trolled by the established dc level. Any attempt to exceed the limit set by the dc level will 
result in a “clipping” (flattening) of the peak region at the high and low end of the output 
signal. In general, therefore, proper amplification design requires that the dc and ac com-
ponents be sensitive to each other’s requirements and limitations. 

 However, it is extremely helpful to realize that: 

  The superposition theorem is applicable for the analysis and design of the dc and ac 
components of a BJT network, permitting the separation of the analysis of the dc and 
ac responses of the system.  

 In other words, one can make a complete dc analysis of a system before considering the 

ac response. Once the dc analysis is complete, the ac response can be determined using a 
completely ac analysis. It happens, however, that one of the components appearing in the 
ac analysis of BJT networks will be determined by the dc conditions, so there is still an 
important link between the two types of analysis.  

   5.3 

 BJT  TRANSISTOR  MODELING 

 ● 

 The key to transistor small-signal analysis is the use of the equivalent circuits (models) to 
be introduced in this chapter. 

  A model is a combination of circuit elements, properly chosen, that best approximates 
the actual behavior of a semiconductor device under specific operating conditions.  

 Once the ac equivalent circuit is determined, the schematic symbol for the device can 

be replaced by this equivalent circuit and the basic methods of circuit analysis applied to 
determine the desired quantities of the network. 

 In the formative years of transistor network analysis the  hybrid equivalent network   was 

employed the most frequently. Specification sheets included the parameters in their listing, 
and analysis was simply a matter of inserting the equivalent circuit with the listed values. 


background image

The drawback to using this equivalent circuit, however, is that it is  defined for a set of oper-
ating conditions that might not match the actual operating conditions.
  In most cases, this is 
not a serious flaw because the actual operating conditions are relatively close to the chosen 
operating conditions on the data sheets. In addition, there is always a variation in actual 
resistor values and given transistor beta values, so as an approximate approach it was quite 
reliable. Manufacturers continue to specify the hybrid parameter values for a particular 
operating point on their specification sheets. They really have no choice. They want to give 
the user some idea of the value of each important parameter so comparisons can be made 
between transistors, but they really do not know the user’s actual operating conditions. 

 In time the use of the  

e

   model  became the more desirable approach because an impor-

tant parameter of the equivalent circuit was determined by the actual operating conditions 
rather than using a data sheet value that in some cases could be quite different. Unfortu-
nately, however, one must still turn to the data sheets for some of the other parameters of 
the equivalent circuit. The  

e

   model also failed to include a feedback term, which in some 

cases can be important if not simply troublesome. 

 The   

e

   model is really a reduced version of the  hybrid  

p  model  used almost exclusively 

for high-frequency analysis. This model also includes a connection between output and 
input to include the feedback effect of the output voltage and the input quantities. The full 
hybrid model is introduced in  Chapter    9   . 

 Throughout the text the  

e

   model is the model of choice unless the discussion centers 

on the description of each model or a region of examination that predetermines the model 
that should be used. Whenever possible, however, a comparison between models will be 
discussed to show how closely related they really are. It is also important that once you gain 
a proficiency with one model it will carry over to an investigation using a different model, 
so moving from one to another will not be a dramatic undertaking. 

 In an effort to demonstrate the effect that the ac equivalent circuit will have on the 

analysis to follow, consider the circuit of  Fig.   5.3   . Let us assume for the moment that the 
small-signal ac equivalent circuit for the transistor has already been determined. Because 
we are interested only in the ac response of the circuit, all the dc supplies can be replaced 
by a zero-potential equivalent (short circuit) because they determine only the dc (quiescent 
level) of the output voltage and not the magnitude of the swing of the ac output. This is 
clearly demonstrated by  Fig.   5.4   . The dc levels were simply important for determining the 
proper  Q -point of operation. Once determined, the dc levels can be ignored in the ac analy-
sis of the network. In addition, the coupling capacitors  C  

1

  and  C  

2

  and bypass capacitor  C  

3

  

were chosen to have a very small reactance at the frequency of application. Therefore, they, 
too, may for all practical purposes be replaced by a low-resistance path or a short circuit. 
Note that this will result in the “shorting out” of the dc biasing resistor  

E

  . Recall that ca-

pacitors assume an “open-circuit” equivalent under dc steady-state conditions, permitting 
an isolation between stages for the dc levels and quiescent conditions. 

  

255

BJT TRANSISTOR 

MODELING

 FIG. 5.3 

          Transistor  circuit  under  examination  in  this  introductory  discussion.    


background image

BJT AC ANALYSIS

256

 

 It is important as you progress through the modifications of the network to define the ac 

equivalent that the parameters of interest such as    Z

i

Z

o

I

i

,     and   

o

   as  defined  by   Fig.    5.5     be 

carried through properly. Even though the network appearance may change, you want to be 
sure the quantities you find in the reduced network are the same as defined by the original 
network. In both networks the input impedance is defined from base to ground, the input 
current as the base current of the transistor, the output voltage as the voltage from collector 
to ground, and the output current as the current through the load resistor  

C

  . 

I

i

I

o

Z

i

Z

o

V

o

V

i

+

+

 FIG. 5.4 

          The  network  of   Fig.    5.3     following  removal  of  the  dc 

supply and insertion of the short-circuit equivalent 

for the capacitors.    

I

i

Z

i

+

I

o

Z

o

V

i

V

o

+

System

 FIG. 5.5 

          Defining  the  important  parameters 

of any system.    

I

i

I

o

V

i

V

o

+

+

+

+

R

i

R

o

 FIG. 5.6 

          Demonstrating  the  reason  for  the  defined 

directions and polarities.    

 The parameters of  Fig.   5.5    can be applied to any system whether it has one or a thou-

sand components. For all the analysis to follow in this text, the directions of the currents, 
the polarities of the voltages, and the direction of interest for the impedance levels are as 
appearing in  Fig.   5.5   . In other words, the input current  

i

   and output current  

o

   are, by defini-

tion, defined to enter the system. If, in a particular example, the output current is leaving the 
system rather than entering the system as shown in  Fig.   5.5   , a minus sign must be applied. 
The defined polarities for the input and output voltages are also as appearing in  Fig.   5.5   . If 
 

o

   has the opposite polarity, the minus sign must be applied. Note that  

i

   is the impedance 

“looking into” the system, whereas  

o

   is the impedance “looking back into” the system 

from the output side. By choosing the defined directions for the currents and voltages as 
appearing in  Fig.   5.5   , both the input impedance and output impedance are defined as having 
positive values. For example, in  Fig.   5.6    the input and output impedances for a particular 
system are both resistive. For the direction of  

i

   and  

o

   the resulting voltage across the resis-

tive elements will have the same polarity as  

i

   and  

o

  , respectively. If  

o

   had been defined 

as the opposite direction in  Fig.   5.5    a minus sign would have to be applied. For each case 
   Z

i

V

i

>I

i

     and     Z

o

V

o

>I

o

    with positive results if they all have the defined directions and 

polarity of  Fig.   5.5   . If the output current of an actual system has a direction opposite to that 


background image

257

THE  r

e

 TRANSISTOR 

MODEL

of  Fig.   5.5    a minus sign must be applied to the result because  

o

   must be defined as appear-

ing in  Fig.   5.5   . Keep  Fig.   5.5    in mind as you analyze the BJT networks in this chapter. It is 
an important introduction to “System Analysis,” which is becoming so important with the 
expanded use of packaged IC systems. 

  

 If we establish a common ground and rearrange the elements of  Fig.   5.4   ,  R  

1

  and  R  

2

   will 

be in parallel, and  

C

   will appear from collector to emitter as shown in  Fig.   5.7   . Because 

the components of the transistor equivalent circuit appearing in  Fig.   5.7    employ familiar 
components such as resistors and independent controlled sources, analysis techniques 
such as superposition, Thévenin’s theorem, and so on, can be applied to determine the 
desired quantities. 

B

I

i

Z

i

 FIG. 5.7 

          Circuit  of   Fig.    5.4     redrawn  for  small-signal  ac  analysis.    

+

V

i

+

V

be

B

C

E

I

e

I

b

 FIG. 5.8 

          Finding  the  input  equivalent  circuit 

for a BJT transistor.    

 Let us further examine  Fig.   5.7    and identify the important quantities to be determined 

for the system. Because we know that the transistor is an amplifying device, we would 
expect some indication of how the output voltage  

o

   is related to the input voltage  

i

  —

the  voltage gain.  Note in  Fig.   5.7    for this configuration that the  current gain  is defined 
by     A

i

I

o

>I

i

.    

 In summary, therefore, the ac equivalent of a transistor network is obtained by: 

    1.    Setting all dc sources to zero and replacing them by a short-circuit equivalent  
   2.    Replacing all capacitors by a short-circuit equivalent  
   3.    Removing all elements bypassed by the short-circuit equivalents introduced by steps 

1 and 2  

   4.    Redrawing the network in a more convenient and logical form   

 In the sections to follow, a transistor equivalent model will be introduced to complete 

the ac analysis of the network of  Fig.    5.7   . 

 

 

   5.4 

 THE   

e

   TRANSISTOR MODEL 

 ● 

 The   

e

   model for the CE, CB, and CC BJT transistor configurations will now be introduced 

with a short description of why each is a good approximation to the actual behavior of a 
BJT transistor. 

  Common-Emitter  Configuration 

 The equivalent circuit for the common-emitter configuration will be constructed using the 
device characteristics and a number of approximations. Starting with the input side, we find 
the applied voltage  

i

   is equal to the voltage  

be

   with the input current being the base cur-

rent  

b

   as  shown  in   Fig.    5.8   . 

 Recall from  Chapter   3    that because the current through the forward-biased junction of 

the transistor is  

E

  , the characteristics for the input side appear as shown in  Fig.   5.9a    for 

various levels of  

BE

  . Taking the average value for the curves of  Fig.   5.9a    will result in the 

single curve of  Fig.   5.9b   , which is simply that of a forward-biased diode. 

  


background image

BJT AC ANALYSIS

258

 

 For the equivalent circuit, therefore, the input side is simply a single diode with a current 

 

e

  , as shown in  Fig.   5.10   . However, we must now add a component to the network that will 

establish the current  

e

   of   Fig.    5.10     using  the  output  characteristics. 

 If we redraw the collector characteristics to have a constant 

b  as  shown  in   Fig.    5.11    

(another approximation), the entire characteristics at the output section can be replaced by 
a controlled source whose magnitude is beta times the base current as shown in  Fig.   5.11   . 
Because all the input and output parameters of the original configuration are now present, the 
equivalent network for the common-emitter configuration has been established in  Fig.   5.12   . 

   

0

0.7 V

(a)

(b)

I

E

V

BE

Various

values
of V

CB

Average

value

of V

CB

0

0.7 V

I

E

V

BE

 FIG. 5.9 

          Defining  the  average  curve  for  the  characteristics  of   Fig.    5.9a   .    

V

be

+

I

b

I

e

I

c

 FIG. 5.10 

          Equivalent  circuit  for  the  input  side 

of a BJT transistor.    

I

B1

V

CE

0

I

B2

I

B3

I

B4

I

B5

I

B6

Constant 

β

I

C

 FIG. 5.11 

          Constant  

b   characteristics.    

V

be

V

ce

+

+

I

b

I

c

I

b

I

e

β

 FIG. 5.12 

          BJT  equivalent  circuit.    

 The equivalent model of  Fig.   5.12    can be awkward to work with due to the direct con-

nection between input and output networks. It can be improved by first replacing the diode 
by its equivalent resistance as determined by the level of  

E

  ,  as  shown  in   Fig.    5.13   .  Recall 

from  Section   1.8    that the diode resistance is determined by    r

D

= 26 mV>I

D

.     Using  the  sub-

script  e  because the determining current is the emitter current will result in    r

e

= 26 mV>I

E

.    

 Now, for the input side:  

   Z

i

=

V

i

I

b

=

V

be

I

b

    

 Solving  for     V

be

:     

   V

be

I

e

r

e

= (I

c

I

b

)r

e

= (bI

b

I

b

)r

e

       

 

 

= (b + 1)I

b

r

e

    

 and  

   Z

i

=

V

be

I

b

=

(

b + 1)I

b

r

e

I

b

   

 

   Z

i

= (b + 1)r

e

⬵ br

e

  

(5.1)

    

V

i

+

V

be

+

Z

i

r

e

I

b

β

I

e

I

b

 FIG. 5.13 

          Defining  the  level  of  Z 

i

 .    


background image

259

THE  r

e

 TRANSISTOR 

MODEL

 The result is that the impedance seen “looking into” the base of the network is a resistor 
equal to beta times the value of  

e

  , as shown in  Fig.   5.14   . The collector output current is 

still linked to the input current by beta as shown in the same figure. 

r

e

β

I

b

I

c

β

I

b

b

e

c

e

 FIG. 5.14 

          Improved  BJT  equivalent  circuit.    

0

V

CEQ

V

A

V

V

CEQ

V

CE  

(V)

Slope = 

1

ro

2

Slope = 

1

ro

1

ΔI

C

ΔI

C

ΔV

CE

ΔV

CE

I

C

(mA)

I

CQ

 FIG. 5.15 

         Defining the Early voltage and the output impedance of a transistor.    

 The equivalent circuit has therefore been defined for the ideal characteristics of  Fig.   5.11   , 

but now the input and output circuits are isolated and only linked by the controlled source—a 
form much easier to work with when analyzing networks. 

  

 

  Early  Voltage 

 We now have a good representation for the input circuit, but aside from the collector out-
put current being defined by the level of beta and  

B

  , we do not have a good representation 

for the output impedance of the device. In reality the characteristics do not have the ideal 
appearance of  Fig.   5.11   . Rather, they have a slope as shown In  Fig.   5.15    that defines the 
output impedance of the device. The steeper the slope, the less the output impedance and 
the less ideal the transistor. In general, it is desirable to have large output impedances to 
avoid loading down the next stage of a design. If the slope of the curves is extended until 
they reach the horizontal axis, it is interesting to note in  Fig.   5.15    that they will all intersect 
at a voltage called the Early voltage. This intersection was first discovered by James M. 
Early in 1952. As the base current increases the slope of the line increases, resulting in an 
increase in output impedance with increase in base and collector current. For a particular 
collector and base current as shown in  Fig.   5.15   , the output impedance can be found using 
the following equation: 

 

 

   r

o

=

V

I

=

V

A

V

CE

Q

I

C

Q

  

(5.2)    


background image

BJT AC ANALYSIS

260

 

 Typically, however, the Early voltage is sufficiently large compared with the applied 
collector-to-emitter voltage to permit the following approximation. 

 

   r

o

V

A

I

C

Q

  

(5.3)    

 Clearly, since  

A

   is a fixed voltage, the larger the collector current, the less the output 

impedance. 

 For situations where the Early voltage is not available the output impedance can be found 

from the characteristics at any base or collector current using the following equation: 

 

   Slope

=

y
x

=

I

C

V

CE

=

1

r

o

   

 and  

   r

o

=

V

CE

I

C

  

(5.4)    

 For the same change in voltage in  Fig.   5.15    the resulting change in current 

¢ 

C

   is signifi-

cantly less for    r

o

2

     than     r

o

1

,     resulting  in     r

o

2

    being much larger than    r

o

1

.    

 In situations where the specification sheets of a transistor do not include the Early volt-

age or the output characteristics, the output impedance can be determined from the hybrid 
parameter  

oe

   that is normally plotted on every specification sheet. It is a quantity that will 

be described in detail in  Section    5.19   . 

 In any event, an output impedance can now be defined that will appear as a resistor in 

parallel with the output as shown in the equivalent circuit of  Fig.    5.16   . 

 

 FIG. 5.16 

         r 

e

  model for the common-emitter transistor configuration 

including effects of r 

o

 .    

 The equivalent circuit of  Fig.   5.16    will be used throughout the analysis to follow for the 

common-emitter configuration. Typical values of beta run from 50 to 200, with values of 

e

   typically running from a few hundred ohms to a maximum of 6 k

⍀ to 7 k⍀. The output 

resistance  r  is typically in the range of    40 k

⍀ to 50 k⍀.     

  Common-Base  Configuration 

 The common-base equivalent circuit will be developed in much the same manner as 
applied to the common-emitter configuration. The general characteristics of the input and 
output circuit will generate an equivalent circuit that will approximate the actual behavior 
of the device. Recall for the common-emitter configuration the use of a diode to represent 
the connection from base to emitter. For the common-base configuration of  Fig.   5.17a    the 
 pnp  transistor employed will present the same possibility at the input circuit. The result is 
the use of a diode in the equivalent circuit as shown in  Fig.   5.17b   . For the output circuit, if 
we return to  Chapter   3    and review  Fig.   3.8   , we find that the collector current is related to 
the emitter current by alpha 

a. In this case, however, the controlled source defining the 

collector current as inserted in  Fig.   5.17b    is opposite in direction to that of the controlled 
source of the common-emitter configuration. The direction of the collector current in the 
output circuit is now opposite that of the defined output current. 

 


background image

 For the ac response, the diode can be replaced by its equivalent ac resistance determined 

by     r

e

= 26 mV>I

E

    as shown in  Fig.   5.18   . Take note of the fact that the emitter current 

continues to determine the equivalent resistance. An additional output resistance can be 
determined from the characteristics of  Fig.   5.19    in much the same manner as applied to the 
common-emitter configuration. The almost horizontal lines clearly reveal that the output 
resistance  

o

   as appearing in  Fig.   5.18    will be quite high and certainly much higher than that 

for the typical common-emitter configuration. 

 The network of  Fig.   5.18    is therefore an excellent equivalent circuit for the analysis of 

most common-base configurations. It is similar in many ways to that of the common-emitter 
configuration. In general, common-base configurations have very low input impedance 
because it is essentially simply  

e

  . Typical values extend from a few ohms to perhaps 50 

The output impedance  

o

   will typically extend into the megohm range. Because the output 

current is opposite to the defined  

o

   direction, you will find in the analysis to follow that 

there is no phase shift between the input and output voltages. For the common-emitter 
configuration there is a 180 °  phase shift. 

  

 

(a)                                                                                                            (b) 

V

i

+

V

o

+

Z

o

Z

i

Z

i

Z

o

I

i

I

e

I

c

I

o

I

c

I

o

I

i

I

e

 FIG. 5.17 

         (a) Common-base BJT transistor; (b) equivalent circuit for configuration of (a).    

261

I

i

Z

i

Z

o

r

o

I

e

I

c

V

o

+

V

i

+

I

o

 FIG. 5.18 

          Common  base  r 

e

   equivalent  circuit.    

0

1

2

3

4

I

C

(mA)

V

CB

Slope = 

1

ro

I

E

 = 4 mA

I

E

 = 3 mA

I

E

 = 2 mA

I

E

 = 1 mA

I

E

 = 0 mA

 FIG. 5.19 

          Defining  Z 

o

 .    


background image

BJT AC ANALYSIS

262

 

  Common-Collector  Configuration 

 For the common-collector configuration, the model defined for the common-emitter configu-
ration of  Fig.   5.16    is normally applied rather than defining a model for the common-collector 
configuration. In subsequent chapters, a number of common-collector configurations will be 
investigated, and the effect of using the same model will become quite apparent.  

  npn versus pnp 

 The dc analysis of npn and pnp configurations is quite different in the sense that the currents 
will have opposite directions and the voltages opposite polarities. However, for an ac analy-
sis where the signal will progress between positive and negative values, the ac equivalent 
circuit will be the same.   

   5.5 

 COMMON-EMITTER  FIXED-BIAS 
CONFIGURATION 

 ● 

 The transistor models just introduced will now be used to perform a small-signal ac analy-
sis of a number of standard transistor network configurations. The networks analyzed rep-
resent the majority of those appearing in practice. Modifications of the standard 
configurations will be relatively easy to examine once the content of this chapter is reviewed 
and understood. For each configuration, the effect of an output impedance is examined for 
completeness. 

 The computer analysis section includes a brief description of the transistor model em-

ployed in the PSpice and Multisim software packages. It demonstrates the range and depth 
of the available computer analysis systems and how relatively easy it is to enter a complex 
network and print out the desired results. The first configuration to be analyzed in detail is 
the common-emitter  fixed-bias  network of  Fig.   5.20   . Note that the input signal  

i

   is applied 

to the base of the transistor, whereas the output  

o

   is off the collector. In addition, recognize 

that the input current  

i

   is not the base current, but the source current, and the output current 

 

o

   is the collector current. The small-signal ac analysis begins by removing the dc effects 

of  

CC

   and replacing the dc blocking capacitors  C  

1

  and  C  

2

  by short-circuit equivalents, 

resulting in the network of  Fig.    5.21   . 

R

B

R

C

V

 

V

CC

C

2  

I

 

Z

 

Z

 

C

1  

V

 

I

 

B

C

E

 FIG. 5.20 

          Common-emitter  fixed-bias  configuration.    

V

 

I

i

I

 

R

 

R

B 

B

C

E

Z

 

Z

 

V

 

 FIG. 5.21 

          Network  of   Fig.    5.20     following  the  removal 

of the effects of V 

CC

 ,  C 

1

 , and C 

2

 .    

 Note in  Fig.   5.21    that the common ground of the dc supply and the transistor emitter 

terminal permits the relocation of  

B

   and  

C

   in parallel with the input and output sections 

of the transistor, respectively. In addition, note the placement of the important network 
parameters  

i

  ,   

o

  ,   

i

  , and  

o

   on the redrawn network. Substituting the  

e

   model for the 

common-emitter configuration of  Fig.    5.21     results  in  the  network  of   Fig.    5.22   . 

   

 The next step is to determine 

b,  

e

  , and  

o

  . The magnitude of 

b is typically obtained 

from a specification sheet or by direct measurement using a curve tracer or transistor 


background image

263

COMMON-EMITTER 

FIXED-BIAS  

CONFIGURATION

testing instrument. The value of  

e

   must be determined from a dc analysis of the system, 

and the magnitude of  

o

   is typically obtained from the specification sheet or characteristics. 

Assuming that 

b,  

e

  , and  

o

   have been determined will result in the following equations for 

the important two-port characteristics of the system. 

      

i

   

 Figure    5.22     clearly  shows  that  

  

 

   Z

i

R

B

7

br

e

 

  

ohms  

(5.5)

    

 For the majority of situations  

B

   is greater than 

e

   by more than a factor of 10 (recall 

from the analysis of parallel elements that the total resistance of two parallel resistors is 
always less than the smallest and very close to the smallest if one is much larger than the 
other), permitting the following approximation: 

 

   Z

i

 

 

br

e  

  RB Ú 10br

e

 

   

ohms  

(5.6)     

  

o

     

  Recall that the output impedance of any system is defined as the impedance  

o

   

determined when  

i

  

  0.  For   Fig.    5.22   ,  when   

i

  

  0,     I

i

I

b

= 0,    resulting in an open-

circuit equivalence for the current source. The result is the configuration of  Fig.   5.23   . 
We have 

 

   Z

o

R

C

7

r

o

 

  

ohms  

(5.7)

    

 If     r

o

Ú 10R

C

,     the  approximation     R

C

7

r

o

⬵ R

C

    is frequently applied, and 

 

   Z

o

⬵ R

C  

  ro Ú 10R

C

  

(5.8)

    

 

 

  

v

     

  The  resistors   

o

   and  

C

   are in parallel, and 

 

   V

o

= -bI

b

(R

C

7

r

o

)   

 but  

   I

b

=

V

i

br

e

   

 so  that  

   V

o

= -ba

V

i

br

e

b(R

C

7

r

o

)   

 and  

   A

v

=

V

o

V

i

= -

(R

C

r

o

)

r

e

  

(5.9)    

 If     r

o

Ú 10R

C

,    so that the effect of  

o

   can be ignored, 

 

   A

v

= -

R

C

r

e

  

r

o

Ú10R

C

  

(5.10)    

 Note the explicit absence of 

b in Eqs. (5.9) and (5.10), although we recognize that b must 

be utilized to determine  

e

  .  

+

I

b

I

c

b

c

+

I

b

β  

I

i

I

 

V

 

Z

 

R

 

R

B 

Z

 

V

 

r

e

β

r

 

 FIG. 5.22 

          Substituting  the  r 

e

   model  into  the  network  of   Fig.    5.21   .    

Z

 

R

 

r

 

 FIG. 5.23 

          Determining  Z 

o

  for the network 

of   Fig.    5.22   .    


background image

BJT AC ANALYSIS

264

 

  

Phase Relationship    

  The negative sign in the resulting equation for  

v

   reveals that a 180° 

phase shift occurs between the input and output signals, as shown in  Fig.   5.24   . The is a 
result of the fact that 

b

   establishes a current through  

C

   that will result in a voltage across 

 

C

  , the opposite of that defined by  

o

  . 

 

V

i

R

 

V

CC

R

B

V

 

V

 

V

 

t

t

0

0

 FIG. 5.24 

          Demonstrating  the   180°   phase  shift  between  input  and  output  waveforms.    

12 V

I

o

Z

 

V

 

I

 

Z

 

3 k

Ω

10    F

= 100

β

470 k

Ω

V

 

= 50 kΩ

r

o

μ 

10    F

μ 

 FIG. 5.25 

           Example    5.1   .    

  

Solution:  

 

 

     a.    DC  analysis: 

 

    I

B

=

V

CC

V

BE

R

B

=

12 V

- 0.7 V

470 k

= 24.04 mA

 

 I

E

= (b + 1)I

B

= (101)(24.04 mA) = 2.428 mA

 

 r

e

=

26 mV

I

E

=

26 mV

2.428 mA

10.71 ⍀

    

   b.        

br

e

= (100)(10.71 ) = 1.071 k

    

         Z

i

R

B

7

br

e

= 470 k

7

1.071 k

 = 1.07 k⍀    

   c.       Z

o

R

C

3 k⍀     

   d.       A

v

= -

R

C

r

e

= -

3 k

10.71 

= ⴚ280.11     

  

EXAMPLE 5.1 

  For  the  network  of   Fig.    5.25   : 

     a.    Determine   

e

  .  

    b.    Find   

i

   (with     r

o

=  ).     

    c.    Calculate   

o

   (with     r

o

=  ).     

    d.    Determine   

v

   (with     r

o

=  ).     

   e.    Repeat parts (c) and (d) including    r

o

= 50 k    in all calculations and compare results.   

 


background image

265

VOLTAGE-DIVIDER BIAS

   e.        Z

o

r

o

7

R

C

= 50 k

7

3 k

 = 2.83 k⍀ vs. 3 k     

        A

v

= -

r

o

R

C

r

e

=

2.83 k

10.71 

= ⴚ264.24 vs. -280.11          

   5.6 

 VOLTAGE-DIVIDER  BIAS 

 ● 

 The next configuration to be analyzed is the  voltage-divider   bias  network  of   Fig.    5.26   . 
Recall that the name of the configuration is a result of the voltage-divider bias at the input 
side to determine the dc level of  

B

  . 

 Substituting the  

e

   equivalent circuit results in the network of  Fig.   5.27   . Note the absence 

of  

E

   due to the low-impedance shorting effect of the bypass capacitor,  

E

  . That is, at the 

frequency (or frequencies) of operation, the reactance of the capacitor is so small compared 
to  

E

   that it is treated as a short circuit across  

E

  . When  

CC

   is set to zero, it places one 

end of  R  

1

  and  

C

   at ground potential as shown in  Fig.   5.27   . In addition, note that  R  

1

   and 

 R  

2

  remain part of the input circuit, whereas  

C

   is part of the output circuit. The parallel 

combination of  R  

1

  and  R  

2

  is defined by 

 

   R

 = R

1

7

R

2

=

R

1

R

2

R

1

R

2

  

(5.11)    

     

i

     

  From   Fig.    5.27    

 

   Z

i

R

7

br

e

  

(5.12)

    

  

 

V

CC

C

1  

C

 

V

i

I

 

I

 

R

 

C

2  

Z

o 

R

E

R

2

Z

i

B

C

E

R

1

V

 

 FIG. 5.26 

          Voltage-divider  bias  configuration.    

I

b

β  

I

b 

I

 

R'

I

 

+

+

b

c

e

e

V

 

R

 

r

 

r

e

β

R

2

R

1

V

i

Z

i

Z

 

 FIG. 5.27 

          Substituting  the  r 

e

  equivalent circuit into the ac equivalent network of  Fig.   5.26   .    


background image

BJT AC ANALYSIS

266

 

  

o

     

  From   Fig.    5.27     with   

i

   set to 0 V, resulting in    I

b

= 0 mA     and     bI

b

= 0 mA,    

 

   Z

o

R

C

7

r

o

  

(5.13)    

 If     r

o

Ú 10R

C

,    

 

   Z

o

⬵ R

C  

  ro Ú 10R

C

  

(5.14)

     

  

v

     

  Because   

C

   and  

o

   are in parallel, 

 

   V

o

= -(bI

b

)(R

C

7

r

o

)   

 and  

   I

b

=

V

i

br

e

   

 so  that  

   V

o

= -ba

V

i

br

e

b(R

C

7

r

o

)   

 and  

   A

v

=

V

o

V

i

=

-R

C

r

o

r

e

  

(5.15)

    

 which you will note is an exact duplicate of the equation obtained for the fixed-bias con-
figuration. 

 For     r

o

Ú 10R

C

,    

 

   A

v

=

V

o

V

i

⬵ -

R

C

r

e

  

 

r

o

Ú10R

C

  

(5.16)     

  

Phase Relationship    

  The negative sign of Eq. (5.15) reveals a 180° phase shift between 

 

o

   and  

i

  . 

  

EXAMPLE 5.2 

  For  the  network  of   Fig.    5.28   ,  determine: 

    a.     

e

  .  

   b.     

i

  .  

   c.     

o

      (r

o

=  ).     

   d.       A

v

 (r

o

=  ).     

   e.    The parameters of parts (b) through (d) if    r

o

= 50 k     and  compare  results.   

 

Z

o 

V

i

= 90

β

22 V

6.8 k

Ω

10    F

1.5 k

Ω

8.2 k

Ω

56 k

Ω

Z

 

I

 

I

 

V

 

μ

10    F

μ 

20    F

μ 

 FIG. 5.28 

         Example    5.2   .    


background image

267

CE EMITTER-BIAS 

CONFIGURATION

  

Solution:  

 

 

     a.    DC:  Testing     

bR

E

7 10R

2

,    

 

    (90)(1.5 k

) 7 10(8.2 k)

  

135 k

 7 82 k (satisfied)   

     Using the approximate approach, we obtain 

 

    V

B

=

R

2

R

1

R

2

 V

CC

=

(8.2 k

)(22 V)

56 k

 + 8.2 k

= 2.81 V

 

 V

E

V

B

V

BE

= 2.81 V - 0.7 V = 2.11 V

 

 I

E

=

V

E

R

E

=

2.11 V

1.5 k

= 1.41 mA

 

 r

e

=

26 mV

I

E

=

26 mV

1.41 mA

18.44 ⍀

    

   b.        R

 = R

1

7

R

2

= (56 k)

7

(8.2 k

) = 7.15 k

    

        Z

i

R

7

br

e

= 7.15 k

7

(90)(18.44 

) = 7.15 k

7

1.66 k

   

 

1.35 k

    

   c.       Z

o

R

C

6.8 k⍀     

   d.       A

v

= -

R

C

r

e

= -

6.8 k

18.44 

= ⴚ368.76     

   e.        Z

i

1.35 k

    

        Z

o

R

C

7

r

o

= 6.8 k

7

50 k

 = 5.98 k⍀ vs. 6.8 k

     A

v

= -

R

C

r

o

r

e

= -

5.98 k

18.44 

= ⴚ324.3 vs. -368.76     

 There was a measurable difference in the results for  

o

   and  

v

  , because the condition 

   r

o

Ú 10R

C

     was   not   satisfied.      

   5.7 

 CE  EMITTER-BIAS  CONFIGURATION 

 ● 

 The networks examined in this section include an emitter resistor that may or may not be 
bypassed in the ac domain. We first consider the unbypassed situation and then modify the 
resulting equations for the bypassed configuration. 

  Unbypassed 

 The most fundamental of unbypassed configurations appears in  Fig.   5.29   . The  

e

   equiva-

lent model is substituted in  Fig.   5.30   , but note the absence of the resistance  

o

  . The effect 

of  

o

   is to make the analysis a great deal more complicated, and considering the fact that in 

R

E

R

B

V

i

Z

 

I

 

V

CC

C

2  

Z

 

C

1

I

 

V

 

R

C

 FIG. 5.29 

        CE  emitter-bias  configuration.    

I

b

β  

r

e

β

Z

i

Z

 

R

E

R

B

R

C

Z

b 

b

c

I

 

+

V

 

β

(   

+ 1)

I

=

 I

b 

e

+

V

 

I

b 

I

 

 FIG. 5.30 

        Substituting  the  r 

e

  equivalent circuit into the ac equivalent network of  Fig.   5.29   .    


background image

BJT AC ANALYSIS

268

 

most situations its effect can be ignored, it will not be included in the present analysis. 
However, the effect of  

o

   will be discussed later in this section.    

 Applying Kirchhoff’s voltage law to the input side of  Fig.   5.30    results in 

 

    V

i

I

b

br

e

I

e

R

E

   

 or  

    V

i

I

b

br

e

+ (b + I)I

b

R

E

   

 and the input impedance looking into the network to the right of  

B

   is 

 

   Z

b

=

V

i

I

b

= br

e

+ (b + 1)R

E

   

 The result as displayed in  Fig.   5.31    reveals that the input impedance of a transistor with 

an unbypassed resistor  

E

   is determined by 

 

   Z

b

= br

e

+ (b + 1)R

E

  

(5.17)    

 Because 

b is normally much greater than 1, the approximate equation is 

 

   Z

b

⬵ br

e

+ bR

E

   

 and  

   Z

b

⬵ b(r

e

R

E

)  

(5.18)    

 Because   

E

   is usually greater than  

e

  , Eq. (5.18) can be further reduced to 

 

   Z

b

⬵ bR

E

  

(5.19)

    

  

Z 

i

     

  Returning  to   Fig.    5.30   ,  we  have 

 

   Z

i

R

B

7

Z

b

  

(5.20)

     

  

Z 

o

     

  With   

i

   set to zero,    I

b

= 0,     and     bI

b

    can be replaced by an open-circuit equivalent. 

The result is 

 

   Z

o

R

C

  

(5.21)     

  

A 

v

     

 

 

    I

b

=

V

i

Z

b

   

 and  

    V

o

= -I

o

R

C

= -bI

b

R

C

 

 

= -ba

V

i

Z

b

bR

C

   

 with  

   A

v

=

V

o

V

i

= -

bR

C

Z

b

  

(5.22)    

 Substituting     Z

b

⬵ b(r

e

R

E

)     gives 

 

   A

v

=

V

o

V

i

⬵ -

R

C

r

e

R

E

  

(5.23)    

 and for the approximation    Z

b

⬵ bR

E

,    

 

   A

v

=

V

o

V

i

⬵ -

R

C

R

E

  

(5.24)    

 Note the absence of 

b from the equation for  

v

   demonstrating an independence in variation 

of 

b.  

  

Phase Relationship    

  The negative sign in Eq. (5.22) again reveals a 180° phase shift 

between  

o

   and  

i

  .  

R

E

Z

b

r

e

β

 FIG. 5.31 

          Defining  the  input  impedance  of  a 

transistor with an unbypassed 

 emitter  resistor.    


background image

269

CE EMITTER-BIAS 

CONFIGURATION

  

Effect of  

o

 

     

  The equations appearing below will clearly reveal the additional complexity 

resulting from including  

o

   in the analysis. Note in each case, however, that when certain 

conditions are met, the equations return to the form just derived. The derivation of each 
equation is beyond the needs of this text and is left as an exercise for the reader. Each 
equation can be derived through  careful  application of the basic laws of circuit analysis 
such as Kirchhoff’s voltage and current laws, source conversions, Thévenin’s theorem, 
and so on. The equations were included to remove the nagging question of the effect of  

o

   

on the important parameters of a transistor configuration.  

  

Z 

i

     

 

 

   Z

b

= br

e

+ c

(

b + 1) + R

C

>r

o

1

+ (R

C

R

E

)

>r

o

R

E

  

(5.25)

    

 Because the ratio    R

C

>r

o

    is always much less than    (

b + 1),    

 

   Z

b

⬵ br

e

+

(

b + 1)R

E

1

+ (R

C

R

E

)

>r

o

   

 For     r

o

Ú 10(R

C

R

E

),    

 

   Z

b

⬵ br

e

+ (b + 1)R

E

   

 which compares directly with Eq. (5.17). 

 In other words, if    r

o

Ú 10(R

C

R

E

),    all the equations derived earlier result. Because 

   

b + 1 ⬵ b,    the following equation is an excellent one for most applications: 

 

   Z

b

⬵ b(r

e

R

E

)

  

r

o

Ú10(R

C

+R

E

)

  

(5.26)

     

  

o

     

 

 

   Z

o

R

C

£

r

o

+

b(r

o

r

e

)

1

+

br

e

R

E

§

     

(5.27)

    

 However,     r

o

r

e

,     and 

 

   Z

o

⬵ R

C

储 r

o

£

1

+

b

1

+

br

e

R

E

§

   

 which can be written as 

 

   Z

o

⬵ R

C

储 r

o

£

1

+

1

1

b

+

r

e

R

E

§

   

 Typically     1

>b     and     r

e

>R

E

    are less than one with a sum usually less than one. The result 

is a multiplying factor for  

o

   greater than one. For 

b = 100,     r

e

= 10 ,     and     R

E

= 1 k,    

 

   

1

1

b

+

r

e

R

E

=

1

1

100

+

10 

1000 

=

1

0.02

= 50   

 and  

   Z

o

R

C

7

51r

o

   

 which is certainly simply  

C

  .  Therefore, 

 

   Z

o

⬵ R

C  

Any level of r

o

  

(5.28)    

 which was obtained earlier.  


background image

BJT AC ANALYSIS

270

 

  

v

     

 

 

   A

v

=

V

o

V

i

=

-

bR

C

Z

b

c 1 +

r

e

r

o

d +

R

C

r

o

1

+

R

C

r

o

  

(5.29)

    

 The  ratio     

r

e

r

o

V 1,     and 

 

   A

v

=

V

o

V

i

-

bR

C

Z

b

+

R

C

r

o

1

+

R

C

r

o

   

 For     r

o

Ú 10R

C

,    

 

   A

v

=

V

o

V

i

⬵ -

bR

C

Z

b

  

r

o

Ú10R

C

  

(5.30)    

 as  obtained  earlier.   

  Bypassed 

 If   

E

   of  Fig.   5.29    is bypassed by an emitter capacitor  

E

  , the complete  

e

   equivalent model 

can be substituted, resulting in the same equivalent network as  Fig.   5.22   . Equations (5.5) 
to (5.10) are therefore applicable. 

  

EXAMPLE 5.3 

  For  the  network  of   Fig.    5.32   ,  without   

E

   (unbypassed), determine: 

    a.     

e

  .  

   b.     

i

  .  

   c.     

o

  .  

   d.     

v

  .   

 

470 k

Ω

C

2  

= 120, r

o

 

= 40 kΩ

β

20 V

2.2 k

Ω

0.56 k

Ω

I

 

10    F

 μ   

V

i

C

E

C

1

V

 

Z

 

Z

o

I

o

10    F

 μ   

10    F

 μ   

 FIG. 5.32 

         Example    5.3   .    

  

Solution:  

 

 

     a.    DC: 

 

    I

B

=

V

CC

V

BE

R

B

+ (b + 1)R

E

=

20 V

- 0.7 V

470 k

 + (121)0.56 k

= 35.89 mA

 

 I

E

= (b + 1)I

B

= (121)(35.89 mA) = 4.34 mA

   

  

and  

    r

e

=

26 mV

I

E

=

26 mV

4.34 mA

5.99 ⍀

    


background image

271

CE EMITTER-BIAS 

CONFIGURATION

    b.    Testing  the  condition     r

o

Ú 10(R

C

R

E

),     we  obtain 

 

    40 k

 Ú 10(2.2 k + 0.56 k)

  

40 k

 Ú 10(2.76 k) = 27.6 k (satisfied)   

   

Therefore, 

 

    Z

b

⬵ b(r

e

R

E

)

= 120(5.99  + 560 )

 

 

= 67.92 k

   

 and  

    Z

i

R

B

7

Z

b

= 470 k

7

67.92 k

 

 

59.34 k

    

   c.        Z

o

R

C

2.2 k

     

   d.        r

o

Ú 10R

C

     is  satisfied.  Therefore, 

        A

v

=

V

o

V

i

⬵ -

bR

C

Z

b

= -

(120)(2.2 k

)

67.92 k

   

 

= ⴚ3.89

   

   

compared 

to 

-3.93 using Eq. (5.20):    A

v

⬵ -R

C

>R

E

.        

  

EXAMPLE 5.4 

  Repeat  the  analysis  of   Example    5.3     with   

E

   in place. 

  

Solution:  

 

 

    a.    The dc analysis is the same, and    r

e

= 5.99 .     

   b.     

E

   is “shorted out” by  

E

   for the ac analysis. Therefore, 

 

    Z

i

R

B

7

Z

b

R

B

7

br

e

= 470 k

7

(120)(5.99 

)

 

 

= 470 k

7

718.8 

 ⬵ 717.70 ⍀

    

   c.       Z

o

R

C

2.2 k⍀     

   d.        A

v

= -

R

C

r

e

    

         

= -

2.2 k

5.99 

= ⴚ367.28 (a significant increase)       

  

EXAMPLE 5.5 

  For  the  network  of   Fig.    5.33     (with   

E

   unconnected), determine (using 

appropriate approximations): 

    a.     

e

  .  

   b.     

i

  .  

   c.     

o

  .  

   d.     

v

  .   

 

C

2  

2.2 k

Ω

C

 

Z

o 

0.68 k

Ω

16 V

= 210, r

o

 

= 50 kΩ

β

10  k

Ω

90   k

Ω

C

1

V

 

V

i

Z

 

+

I

o

I

i

 FIG. 5.33 

           Example    5.5   .    


background image

BJT AC ANALYSIS

272

 

  

Solution:  

 

 

     a.    Testing     

bR

E

7 10R

2

,    

 

   (210)(0.68 k

) 7 10(10 k)

 142.8 k

 7 100 k (satisfied)   

   

we 

have 

 

   V

B

=

R

2

R

1

R

2

 V

CC

=

10 k

90 k

 + 10 k

(16 V)

= 1.6 V

 

 V

E

V

B

V

BE

= 1.6 V - 0.7 V = 0.9 V

 

 I

E

=

V

E

R

E

=

0.9 V

0.68 k

= 1.324 mA

 

 r

e

=

26 mV

I

E

=

26 mV

1.324 mA

19.64 ⍀

    

   b.    The ac equivalent circuit is provided in  Fig.   5.34   . The resulting configuration is differ-

ent from  Fig.   5.30    only by the fact that now 

 

   R

B

R = R

1

7

R

2

= 9 k   

 

R'

I

o

2.2 k

Ω

0.68 k

Ω

90 k

Ω

10 k

Ω

V

i

Z

o 

V

 

+

+

Z

 

I

i

 FIG. 5.34 

        The  ac  equivalent  circuit  of   Fig.    5.33   .    

     The testing conditions of    r

o

Ú 10(R

C

R

E

)     and     r

o

Ú 10R

C

    are both satisfied. Using 

the appropriate approximations yields 

 

    Z

b

⬵ bR

E

= 142.8 k

 

 Z

i

R

B

7

Z

b

= 9 k

7

142.8 k

 

 

8.47 k

    

   c.       Z

o

R

C

2.2 k⍀     

   d.       A

v

= -

R

C

R

E

= -

2.2 k

0.68 k

= ⴚ3.24        

  

EXAMPLE 5.6 

  Repeat   Example    5.5     with   

E

   in place. 

  

Solution:  

 

 

    a.    The dc analysis is the same, and    r

e

19.64 ⍀.     

   b.        Z

b

= br

e

= (210)(19.64 ) ⬵ 4.12 k    

        Z

i

R

B

7

Z

b

= 9 k

7

4.12 k

   

 

2.83 k

    

   c.       Z

o

R

C

2.2 k⍀     

   d.       A

v

= -

R

C

r

e

= -

2.2 k

19.64 

= ⴚ112.02 (a significant increase)        

 Another variation of an emitter-bias configuration is shown in  Fig.   5.35   . For the dc 

analysis, the emitter resistance is    R

E

1

R

E

2

,    whereas for the ac analysis, the resistor  

E

   in 

the equations above is simply    R

E

1

     with     R

E

2

     bypassed  by   

E

  . 

 

  


background image

273

EMITTER-FOLLOWER 

CONFIGURATION

   5.8 

 EMITTER-FOLLOWER  CONFIGURATION 

 ● 

 When the output is taken from the emitter terminal of the transistor as shown in  Fig.   5.36   , 
the network is referred to as an  emitter-follower.  The output voltage is always slightly less 
than the input signal due to the drop from base to emitter, but the approximation    A

v

⬵ 1    

is usually a good one. Unlike the collector voltage, the emitter voltage is in phase with the 
signal  

i

  . That is, both  

o

   and  

i

   attain their positive and negative peak values at the same 

time. The fact that  

o

   “follows” the magnitude of  

i

   with an in-phase relationship accounts 

for the terminology emitter-follower. 

 

R

C

C

R

B

V

CC

C

E

C

1  

Z

o

R

E

1

V

 

R

E

2

V

i

Z

 

I

o

I

i

 FIG. 5.35 

        An  emitter-bias  configuration  with  a 

 portion of the emitter-bias resistance 

bypassed in the ac domain.    

R

E

R

B

V

CC

V

o

C

2  

Z

 

C

1  

V

 

B

C

E

Z

o

I

o

I

i

 FIG. 5.36 

        Emitter-follower  configuration.    

 The most common emitter-follower configuration appears in  Fig.   5.36   . In fact, because 

the collector is grounded for ac analysis, it is actually a  common-collector   configuration. 
Other variations of  Fig.   5.36    that draw the output off the emitter with    V

o

⬵ V

i

     will  appear 

later in this section. 

 The emitter-follower configuration is frequently used for impedance-matching pur-

poses. It presents a high impedance at the input and a low impedance at the output, which 
is the direct opposite of the standard fixed-bias configuration. The resulting effect is much 
the same as that obtained with a transformer, where a load is matched to the source imped-
ance for maximum power transfer through the system. 

 Substituting the  

e

   equivalent circuit into the network of  Fig.   5.36    results in the network 

of   Fig.    5.37   .  The  effect  of   

o

   will be examined later in the section. 

 


background image

BJT AC ANALYSIS

274

 

     

i

     

  The input impedance is determined in the same manner as described in the preceding 

section: 

 

   Z

i

R

B

7

Z

b

  

(5.31)    

 with  

   Z

b

= br

e

+ (b + 1)R

E

  

(5.32)

    

 or  

   Z

b

⬵ b(r

e

R

E

)  

(5.33)

    

 and  

   Z

b

⬵ bR

E   R

E

Wr

e

  

(5.34)     

  

o

     

  The output impedance is best described by first writing the equation for the current  

b

  , 

 

   I

b

=

V

i

Z

b

   

 and then multiplying by    (

b + 1)     to  establish   

e

  . That is, 

 

   I

e

= (b + 1)I

b

= (b + 1)

V

i

Z

b

   

 Substituting for  

b

   gives 

 

   I

e

=

(

b + 1)V

i

br

e

+ (b + 1)R

E

   

 or  

   I

e

=

V

i

[

br

e

>(b + 1)] + R

E

   

 but  

   (

b + 1) ⬵ b   

 and  

   

br

e

b + 1

br

e

b

r

e

   

 so  that  

   I

e

V

i

r

e

R

E

  

(5.35)    

 If we now construct the network defined by Eq. (5.35), the configuration of  Fig.   5.38    

results. 

 

 To determine  

o

  ,   

i

   is set to zero and 

 

   Z

o

R

E

7

r

e

  

(5.36)    

R

E

R

B 

c

β

(   

+ 1)

I

=

I

b  

e

r

e

β

b

I

i

V

o

I

o

Z

b

+

Z

 

Z

i

I

b 

V

 

+

I

b

β  

 FIG. 5.37 

        Substituting  the  r 

e

  equivalent circuit into the ac 

 equivalent  network  of   Fig.    5.36   .    

R

E

r

e

V

o

V

 

I

e

Z

 

+

 FIG. 5.38 

        Defining  the  output  impedance  for 

the emitter-follower configuration.    


background image

275

EMITTER-FOLLOWER 

CONFIGURATION

 Because   

E

   is typically much greater than  

e

  , the following approximation is often applied: 

 

   Z

o

⬵ r

e

  

(5.37)

     

  

v

     

   Figure   5.38    can be used to determine the voltage gain through an application of the 

voltage-divider rule: 

 

   V

o

=

R

E

V

i

R

E

r

e

   

 and  

   A

v

=

V

o

V

i

=

R

E

R

E

r

e

  

(5.38)    

 Because   

E

   is usually much greater than  

e

  ,    R

E

r

e

⬵ R

E

     and 

 

   A

v

=

V

o

V

i

⬵ 1  

(5.39)

     

  

Phase Relationship    

  As revealed by Eq. (5.38) and earlier discussions of this section,  

o

   

and  

i

   are in phase for the emitter-follower configuration.  

  

Effect of  

o

 

     

     

  

Z 

i  

   

 

 

   Z

b

= br

e

+

(

b + 1)R

E

1

+

R

E

r

o

  

(5.40)

    

 If the condition    r

o

Ú 10R

E

     is  satisfied, 

 

   Z

b

= br

e

+ (b + 1)R

E

   

 which matches earlier conclusions with 

 

   Z

b

⬵ b(r

e

R

E

)

  r

o

Ú10R

E

  

(5.41)     

  

o

     

 

 

   Z

o

r

o

储 R

E

br

e

(

b + 1)

  

(5.42)    

 Using     

b + 1 ⬵ b,     we  obtain 

 

   Z

o

r

o

7

R

E

7

r

e

   

 and  because     r

o

r

e

,    

 

   Z

o

⬵ R

E

7

r

e  Any  r

o

  

(5.43)

     

  

A 

v

     

 

 

   A

v

=

(

b + 1)R

E

>Z

b

1

+

R

E

r

o

  

(5.44)

    

 If the condition    r

o

Ú 10R

E

    is satisfied and we use the approximation    

b + 1 ⬵ b,     we  find 

 

   A

v

bR

E

Z

b

   


background image

BJT AC ANALYSIS

276

 

 But  

   Z

b

⬵ b(r

e

R

E

)   

 so  that  

   A

v

bR

E

b(r

e

R

E

)

   

 and  

   A

v

R

E

r

e

R

E

  

r

o

Ú10R

E

  

(5.45)    

  

EXAMPLE 5.7 

  For  the  emitter-follower  network  of   Fig.    5.39   ,  determine: 

    a.     

e

  .  

   b.     

i

  .  

   c.     

o

  .  

   d.     

v

  .  

   e.    Repeat parts (b) through (d) with    r

o

= 25 k     and  compare  results.   

 

12 V

3.3 k

Ω

= 100, r

o

 

= ∞ Ω

β

220 k

Ω

V

 

V

o

I

o

Z

i

Z

 

I

i

10    F

μ 

10    F

μ 

R

B

R

E

 FIG. 5.39 

         Example    5.7   .    

  

Solution:  

 

 

    a.        I

B

=

V

CC

V

BE

R

B

+ (b + 1)R

E

    

         

=

12 V

- 0.7 V

220 k

 + (101)3.3 k

= 20.42 mA

     I

E

= (b + 1)I

B

   

 

= (101)(20.42 mA) = 2.062 mA

     r

e

=

26 mV

I

E

=

26 mV

2.062 mA

12.61 ⍀

    

   b.        Z

b

= br

e

+ (b + 1)R

E

    

         

= (100)(12.61 ) + (101)(3.3 k)

   

 

= 1.261 k + 333.3 k

   

 

= 334.56 k ⬵ bR

E

     Z

i

R

B

7

Z

b

= 220 k

7

334.56 k

   

 

132.72 k⍀    

   c.        Z

o

R

E

7

r

e

= 3.3 k

7

12.61 

    

         

12.56 ⍀ ⬵ r

e

    

   d.        A

v

=

V

o

V

i

=

R

E

R

E

r

e

=

3.3 k

3.3 k

 + 12.61 

    

         

0.996 1    


background image

277

COMMON-BASE 

CONFIGURATION

    e.    Checking  the  condition     r

o

Ú 10R

E

,     we  have 

 

   25 k

 Ú 10(3.3 k) = 33 k   

     which is  not  satisfied. Therefore, 

 

    Z

b

= br

e

+

(

b + 1)R

E

1

+

R

E

r

o

= (100)(12.61 ) +

(100

+ 1)3.3 k

1

+

3.3 k

25 k

 

 

= 1.261 k + 294.43 k

 

 

= 295.7 k

   

 with  

     Z

i

R

B

7

Z

b

= 220 k

7

295.7 k

 

 

126.15 k⍀ vs. 132.72 k obtained earlier

 

 Z

o

R

E

7

r

e

12.56 ⍀ as obtained earlier

 

 A

v

=

(

b + 1)R

E

>Z

b

c 1 +

R

E

r

o

d

=

(100

+ 1)(3.3 k)>295.7 k

c 1 +

3.3 k

25 k

d

 

0.996 1   

     matching  the  earlier  result.     

 In general, therefore, even though the condition    r

o

Ú 10R

E

    is not satisfied, the results 

for  

o

   and  

v

   are the same, with  

i

   only slightly less. The results suggest that for most ap-

plications a good approximation for the actual results can be obtained by simply ignoring 
the effects of  

o

   for this configuration. 

 The network of  Fig.   5.40    is a variation of the network of  Fig.   5.36   , which employs 

a voltage-divider input section to set the bias conditions. Equations (5.31) to (5.34) are 
changed only by replacing  

B

   by     R

 = R

1

7

R

2

.    

 The network of  Fig.   5.41    also provides the input/output characteristics of an emitter-

follower, but includes a collector resistor  

C

  . In this case  

B

   is again replaced by the parallel 

combination of  R  

1

  and  R  

2

 . The input impedance  

i

   and output impedance  

o

   are unaffected 

by  

C

   because it is not reflected into the base or emitter equivalent networks. In fact, the 

only effect of  

C

   is to determine the  Q -point of operation.      

V

 

V

o

R

E

V

CC

C

1  

C

2  

R

1

I

o

Z

i

Z

 

I

i

R

2

 FIG. 5.40 

        Emitter-follower  configuration  with  a 

voltage-divider biasing arrangement.    

R

 

V

 

V

o

R

E

V

CC

C

1  

C

2  

R

1

I

o

Z

i

Z

 

R

2

 FIG. 5.41 

        Emitter-follower  configuration  with 

a collector resistor R 

C

 .    

   5.9 

 COMMON-BASE  CONFIGURATION 

 ● 

 The common-base configuration is characterized as having a relatively low input and a high 
output impedance and a current gain less than 1. The voltage gain, however, can be quite 
large. The standard configuration appears in  Fig.   5.42   , with the common-base  

e

   equivalent 

model substituted in  Fig.   5.43   . The transistor output impedance  

o

   is not included for the 


background image

common-base configuration because it is typically in the megohm range and can be ignored 
in parallel with the resistor  

C

  .   

     

i

     

 

 

   Z

i

R

E

7

r

e

  

(5.46)     

  

Z 

o

     

 

 

   Z

o

R

C

  

(5.47)

     

  

A 

v

     

 

 

   V

o

= -I

o

R

C

= -(-I

c

)R

C

= aI

e

R

C

   

 with  

   I

e

=

V

i

r

e

   

 so  that  

   V

o

= aa

V

i

r

e

bR

C

   

 and  

   A

v

=

V

o

V

i

=

aR

C

r

e

R

C

r

e

  

(5.48)

     

  

A 

i

     

  Assuming  that     R

E

r

e

     yields 

    

I

e

I

i

   

 and  

   I

o

= -aI

e

= -aI

i

   

 with  

   A

i

=

I

o

I

i

= -a ⬵ -1  

(5.49)

     

  

Phase Relationship    

  The fact that  

v

   is a positive number shows that  

o

   and  

i

   are in 

phase for the common-base configuration.  

  

Effect of  

o

 

     

  For the common-base configuration,    r

o

= 1>h

ob

    is typically in the megohm 

range and sufficiently larger than the parallel resistance  

C

   to permit the approximation 

   r

o

7

R

C

⬵ R

C

.    

  

EXAMPLE 5.8 

  For  the  network  of   Fig.    5.44   ,  determine: 

    a.     

e

  .  

   b.     

i

  .  

   c.     

o

  .  

   d.     

v

  .  

   e.     

i

  .   

 

E

V

i

R

 

R

E

V

CC

C

I

o

Z

i

Z

 

+

V

o

+

I

i

I

e

I

c

B

EE

V

+

+

 FIG. 5.42 

        Common-base  configuration.    

Z

 

V

o

+

R

 

I

o

V

i

+

R

 

e

c

Z

i

I

i

I

e 

 r

e

α

I

e

I

c

 FIG. 5.43 

        Substituting  the  r 

e

  equivalent circuit into the ac equivalent network 

of   Fig.    5.44   .    

I

e

I

i

Z

o

Z

i

1 k

Ω

2 V

V

o

+

10    F

μ

10    F

μ

= 0.98

α

5 k

Ω

8 V

I

o

V

i

I

i

+

r

o

 

= 1 M Ω

+

+

R

E

R

C

 FIG. 5.44 

         Example    5.8   .    

278


background image

279

COLLECTOR FEEDBACK 

CONFIGURATION

  

Solution:  

 

 

    a.        I

E

=

V

EE

V

BE

R

E

=

2 V

- 0.7 V

1 k

=

1.3 V

1 k

= 1.3 mA    

        r

e

=

26 mV

I

E

=

26 mV

1.3 mA

20 ⍀

    

   b.        Z

i

R

E

7

r

e

= 1 k

7

20 

    

         

19.61 ⍀ ⬵ r

e

    

   c.        Z

o

R

C

5 k

     

   d.        A

v

R

C

r

e

=

5 k

20 

250     

   e.        A

i

= ⴚ0.98 ⬵ -1           

   5.10 

 COLLECTOR  FEEDBACK  CONFIGURATION 

 ● 

 The collector feedback network of  Fig.   5.45    employs a feedback path from collector to 
base to increase the stability of the system as discussed in  Section   4.6   . However, the sim-
ple maneuver of connecting a resistor from base to collector rather than base to dc supply 
has a significant effect on the level of difficulty encountered when analyzing the network. 

 Some of the steps to be performed below are the result of experience working with 

such configurations. It is not expected that a new student of the subject would choose 
the sequence of steps described below without taking a wrong step or two. Substituting the 
equivalent circuit and redrawing the network results in the configuration of  Fig.   5.46   . The 
effects of a transistor output resistance  

o

   will be discussed later in the section.   

B

E

R

 

R

F

V

CC

C

2  

C

1

V

o

I

 

V

i

C

I

o

Z

i

Z

o

 FIG. 5.45 

        Collector  feedback  configuration.    

Z

o

R

 

I

c

B

I'

R

 

I

 

r

e

β

I

b

I

i

+

Z

i

V

o

V

i

+

+

C

I

b

β  

 FIG. 5.46 

        Substituting  the  r 

e

  equivalent circuit into the ac 

 equivalent  network  of   Fig.    5.45   .    

     

Z 

i

     

 

 

    I

o

I + bI

b

   

 and  

     I

 =

V

o

V

i

R

F

   

 but  

    V

o

= -I

o 

R

C

= -(I + bI

b

)R

C

   

 with  

     V

i

I

b

br

e

   

 so  that  

   I

 = -

(I

 + bI

b

)R

C

I

b

br

e

R

F

= -

I

R

C

R

F

-

bI

b 

R

C

R

F

-

I

b

br

e

R

F

   

 which when rearranged in the following: 

 

   I

a1 +

R

C

R

F

b = -bI

b

(R

C

r

e

)

R

F

   


background image

BJT AC ANALYSIS

280

 

 and  finally,  

   I

 = -bI

b

(R

C

r

e

)

R

C

R

F

   

 Now      Z

i

=

V

i

I

i

:   

 and  

    I

i

I

b

I = I

b

+ bI

b

 

(R

C

r

e

)

R

C

R

F

   

 or  

    I

i

I

b

a1 + b 

(R

C

r

e

)

R

C

R

F

b

   

 Substituting for  

i

   in the above equation for  

i

   leaves 

 

   Z

i

=

V

i

I

i

=

I

b

br

e

I

b

a1 + b 

(R

C

r

e

)

R

C

R

F

b

=

br

e

1

+ b 

(R

C

r

e

)

R

C

R

F

   

 Since     R

C

r

e

    

 

   Z

i

=

br

e

1

+

bR

C

R

C

R

F

   

 or  

   Z

i

=

r

e

1

b

+

R

C

R

C

R

F

  

(5.50)     

  

Z 

o

     

  If we set  

i

   to zero as required to define  

o

  , the network will appear as shown in  Fig.   5.47   . 

The effect of 

e

   is removed, and  

F

   appears in parallel with  

C

   and 

 

   Z

o

⬵ R

C

7

R

F

  

(5.51)    

 

 

Z

o

R

F

R

C

= 0 A

V

i  

=  0

I

= 0 A

βr

e

βI

b

 FIG. 5.47 

        Defining  Z 

o

  for the collector feedback configuration.    

  

A 

v

     

 

 

    V

o

= -I

o 

R

C

= -(I + bI

b

)R

C

 

 

= - a-bI

b

(R

C

r

e

)

R

C

R

F

+ bI

b

bR

C

   

 or  

    V

o

= -bI

b

a1 -

(R

C

r

e

R

C

R

F

bR

C

   

 Then 

 

    A

v

=

V

o

V

i

=

-bI

b

a1 -

(R

C

r

e

)

R

C

R

F

bR

C

br

e

 I

b

 

 

= - a1 -

(R

C

r

e

)

R

C

R

F

b

R

C

r

e

   

 For     R

C

r

e

    

 

   A

v

= - a1 -

R

C

R

C

R

F

b

R

C

r

e

   


background image

281

COLLECTOR FEEDBACK 

CONFIGURATION

 or  

    A

v

= -

(R

C

R

F

R

C

)

R

C

R

F

R

C

r

e

   

 and  

    A

v

= - a

R

F

R

C

R

F

b

R

C

r

e

  

(5.52)    

 For     R

F

R

C

    

 

   A

v

⬵ -

R

C

r

e

  

(5.53)

     

  

Phase Relationship    

  The negative sign of Eq. (5.52) indicates a 180° phase shift between 

 

o

   and  

i

  .  

  

Effect of  r 

o

 

     

     

  

Z 

i

     

  A complete analysis without applying approximations results in 

 

   Z

i

=

1

+

R

C

r

o

R

F

1

br

e

+

1

R

F

+

R

C

储 r

o

br

e 

R

F

+

R

C

储 r

o

R

F

r

e

  

(5.54)    

 Applying the condition    r

o

Ú 10R

C

,     we  obtain 

 

   Z

i

=

1

+

R

C

R

F

1

br

e

+

1

R

F

+

R

C

br

e 

R

F

+

R

C

R

F

r

e

=

r

e

c 1 +

R

C

R

F

d

1

b

+

1

R

F

r

e

+

R

C

b

R

C

d

   

 Applying     R

C

r

e

     and     

R

C

b

,    

 

   Z

i

r

e

c 1 +

R

C

R

F

d

1

b

+

R

C

R

F

=

r

e

c

R

F

R

C

R

F

d

R

F

+ bR

C

bR

F

=

r

e

1

b

a

R

F

R

F

R

C

b +

R

C

R

C

R

F

   

 but,  since     R

F

 typically

R

C

R

F

R

C

⬵ R

F

     and     

R

F

R

F

R

C

= 1    

 

   Z

i

r

e

1

b

+

R

C

R

C

R

  r

o

WR

C

R

F

7R

C

  

(5.55)

    

 as obtained earlier.  

  

o

     

  Including   

o

   in parallel with  

C

   in   Fig.    5.47     results  in 

 

   Z

o

r

o

7

R

C

7

R

F

  

(5.56)    

 For     r

o

Ú 10R

C

,    

 

   Z

o

⬵ R

C

7

R

F  r

o

Ú10R

C

  

(5.57)

    

 as obtained earlier. For the common condition of    R

F

R

C

,    

 

   Z

o

⬵ R

C

  

r

o

Ú10R

C

,R

F

WR

C

  

(5.58)     


background image

BJT AC ANALYSIS

282

 

  

v

     

 

    

A

v

= - a

R

F

R

C

储 r

o

R

F

b

R

C

储 r

o

r

e

  

(5.59)

    

 For     r

o

Ú 10R

C

,    

    

A

v

⬵ - a

R

F

R

C

R

F

b

R

C

r

e

  

r

o

Ú10R

C

  

(5.60

)    

 and  for     R

F

R

C

    

    

A

v

⬵ -

R

C

r

e

  

r

o

Ú10R

C

R

F

ÚR

C

  

(5.61)    

 as obtained earlier. 

  

EXAMPLE 5.9 

  For  the  network  of   Fig.    5.48.     determine: 

    a.      

e

  .  

    b.       

i

  .  

    c.       

o

  .  

    d.       

v

  .  

   e.    Repeat parts (b) through (d) with    r

o

= 20 k     and  compare  results.         

 

9 V

10 

μF

I

o 

V

o

Z

o

V

i

I

 

Z

 

10 

μF

= 200, r

o

 

= ∞ Ω

β

2.7 k

Ω

180 k

Ω

 FIG. 5.48 

           Example    5.9   .    

  

Solution:  

 

 

    a.        I

B

=

V

CC

V

BE

R

F

+ bR

C

=

9 V

- 0.7 V

180 k

 + (200)2.7 k

    

    

 

= 11.53 mA

 

 I

E

= (b + 1)I

B

= (201)(11.53 mA) = 2.32 mA

 

 r

e

=

26 mV

I

E

=

26 mV

2.32 mA

11.21 ⍀    

   b.        Z

i

=

r

e

1

b

+

R

C

R

C

R

F

=

11.21 

1

200

+

2.7 k

182.7 k

=

11.21 

0.005

+ 0.0148

    

    

 

=

11.21 

0.0198

566.16 ⍀    

   c.       Z

o

R

C

7

R

F

= 2.7 k

7

180 k

 = 2.66 k⍀     

   d.       A

v

= -

R

C

r

e

= -

2.7 k

11.21 

= ⴚ240.86     


background image

283

COLLECTOR FEEDBACK 

CONFIGURATION

   e.       Z

i

:    

The  condition     r

o

Ú 10R

C

     is   not  satisfied. Therefore, 

    Z

i

=

1

+

R

C

r

o

R

F

1

br

e

+

1

R

F

+

R

C

兩兩r

o

br

e

R

F

+

R

C

兩兩r

o

R

F

r

e

=

1

+

2.7 k

 7 20 k

180 k

1

(200)(11.21)

+

1

180 k

+

2.7 k

 兩兩20 k

(200)(11.21 

)(180 k)

+

2.7 k

 兩兩20 k

(180 k

)(11.21 )

 

=

1

+

2.38 k

180 k

0.45

* 10

-3

+ 0.006 * 10

-3

+ 5.91 * 10

-6

+ 1.18 * 10

-3

=

1

+ 0.013

1.64

* 10

-3

 

617.7 ⍀ vs. 566.16  above

    

       Z 

o

  : 

    

Z

o

r

o

7

R

C

7

R

F

= 20 k

7

2.7 k

7

180 k

   

     

2.35 k⍀     vs.     2.66 k     above 

    

v

  : 

    

 

= - a

R

F

R

C

储 r

o

R

F

b

R

C

储 r

o

r

e

= - c

180 k

2.38 k

 + 180 k

d

2.38 k

11.21

 

 

= -

3

0.987

4

212.3

 

 

= ⴚ209.54       

 For the configuration of  Fig.   5.49   , Eqs. (5.61) through (5.63) determine the variables of 

interest. The derivations are left as an exercise at the end of the chapter. 

 

 

V

i

I

 

R

F

R

 

R

E

C

2  

C

V

CC

I

o 

V

o

Z

o

Z

 

 FIG. 5.49 

          Collector  feedback  configuration  with  an  emitter  resistor  R 

E

 .    

  

i  

   

 

    

Z

i

R

E

c

1

b

+

(R

E

R

C

)

R

F

d

  

(5.62)

     

  

o

     

 

    

Z

o

R

C

7

R

F

  

(5.63)     

  

v

     

 

    

A

v

⬵ -

R

C

R

E

  

(5.64)       


background image

BJT AC ANALYSIS

284

 

   5.11 

 COLLECTOR  DC  FEEDBACK  CONFIGURATION 

 ● 

 The network of  Fig.   5.50    has a dc feedback resistor for increased stability, yet the capacitor 
 C  

3

  will shift portions of the feedback resistance to the input and output sections of the net-

work in the ac domain. The portion of  

F

   shifted to the input or output side will be deter-

mined by the desired ac input and output resistance levels. 

 

V

i

I

 

C

3

R

F

1

R

 

C

V

CC

R

F

2

C

2  

I

o 

V

o

Z

o

Z

 

 FIG. 5.50 

          Collector  dc  feedback  configuration.    

R'

I

b

V

i

I

i

+

r

e

β

I

o 

R

 

V

o

Z

o

R

F

2

r

o

R

F

1

Z

 

+

I

b

β  

 FIG. 5.51 

          Substituting  the  r 

e

  equivalent circuit into the ac equivalent network of  Fig.   5.50   .    

 At the frequency or frequencies of operation, the capacitor will assume a short-circuit 

equivalent to ground due to its low impedance level compared to the other elements of the 
network. The small-signal ac equivalent circuit will then appear as shown in  Fig.    5.51   . 

 

     

i

     

 

    

Z

i

R

F

1

7

br

e

  

(5.65)     

  

o

     

 

    

Z

o

R

C

7

R

F

2

7

r

o

  

(5.66)

    

 For     r

o

Ú 10R

C

,    

    

Z

o

⬵ R

C

7

R

F

2

  

r

o

Ú10R

C

  

(5.67)     

  

v

     

 

   R

 = r

o

7

R

F

2

7

R

C

   

 and  

   V

o

= -bI

b

R

   

 but  

   I

b

=

V

i

br

e

   


background image

285

COLLECTOR 

DC FEEDBACK 

CONFIGURATION

 and  

   V

o

= -b

V

i

br

e

R

   

 so  that 

    

A

v

=

V

o

V

i

= -

r

o

R

F

2

R

C

r

e

  

(5.68)    

 For     r

o

Ú 10R

C

,    

    

A

v

=

V

o

V

i

⬵ -

R

F

2

R

C

r

e

 

  

r

o

Ú10R

C

  

(5.69)

     

  

Phase Relationship    

  The negative sign in Eq. (5.68) clearly reveals a 180° phase shift 

between input and output voltages. 

  

EXAMPLE 5.10 

  For  the  network  of   Fig.    5.52   ,  determine: 

    a.     

e

  .  

   b.     

i

  .  

   c.     

o

  .  

   d.     

v

  .  

   e.     

o

  

 if  

i

  

  2  mV   

 

12 V

10 

μF

3 k

Ω

120 k

Ω

= 140, r

o

 

= 30 k Ω

β

0.01 

μF

68 k

Ω

V

i

I

 

10 

μF

I

o 

V

o

Z

o

Z

 

 FIG. 5.52 

           Example    5.10   .    

  

Solution:  

 

 

     a.    DC:      I

B

=

V

CC

V

BE

R

F

+ bR

C

    

    

 

=

12 V

- 0.7 V

(120 k

 + 68 k) + (140)3 k

 

 

=

11.3 V

608 k

= 18.6 mA

 

 I

E

= (b + 1)I

B

= (141)(18.6 mA)

 

 

= 2.62 mA

 

 r

e

=

26 mV

I

E

=

26 mV

2.62 mA

9.92 ⍀    

   b.       

br

e

= (140)(9.92 ) = 1.39 k    

     The ac equivalent network appears in  Fig.   5.53   . 

    

 Z

i

R

F

1

7

br

e

= 120 k

7

1.39 k

 

 

⬵ 1.37 k⍀   

 

 


background image

BJT AC ANALYSIS

286

 

    c.    Testing  the  condition     r

o

Ú 10R

C

,     we  find 

    

30 k

 Ú 10(3 k) = 30 k   

     which is satisfied through the equals sign in the condition. Therefore, 

    

 Z

o

⬵ R

C

7

R

F

2

= 3 k

7

68 k

 

 

2.87 k⍀    

   d.        r

o

Ú 10R

C

;     therefore, 

    

 A

v

⬵ -

R

F

2

R

C

r

e

= -

68 k

7 3 k

9.92 

 

 

⬵ -

2.87 k

9.92 

 

 

⬵ ⴚ289.3    

   e.       

兩 A

v

兩 = 289.3 =

V

o

V

i

    

    

V

o

= 289.3V

i

= 289.3(2 mV) = 0.579 V          

   5.12 

 EFFECT  OF   

L

   AND  

S

   

 ● 

 All the parameters determined in the last few sections have been for an unloaded amplifier 
with the input voltage connected directly to a terminal of the transistor. In this section the 
effect of applying a load to the output terminal and the effect of using a source with an 
internal resistance will be investigated. The network of  Fig.   5.54a    is typical of those inves-
tigated in the previous section. Because a resistive load was not attached to the output ter-
minal, the gain is commonly referred to as the no-load gain and given the following 
notation: 

    

A

v

NL

=

V

o

V

i

  

(5.70)    

 In  Fig.   5.54b    a load has been added in the form of a resistor  

L

  , which will change the 

overall gain of the system. This loaded gain is typically given the following notation: 

    

             A

v

L

=

V

o

V

i

  

with R

L

  

(5.71)    

 In  Fig.   5.54c    both a load and a source resistance have been introduced, which will have 

an additional effect on the gain of the system. The resulting gain is typically given the fol-
lowing notation: 

    

                     A

v

s

=

V

o

V

s

  

with R

L

 and R

s

  

(5.72)    

 

 The analysis to follow will show that: 

  The loaded voltage gain of an amplifier is always less than the no-load gain.  

I

b

V

i

I

i

+

r

e

β

V

o

+

3 k

Ω

1.395 k

Ω

68 k

Ω

r

o

30 k

Ω

140 I

b

120 k

Ω

I

b

β  

I

o 

Z

o

Z

 

 FIG. 5.53 

          Substituting  the  r 

e

  equivalent circuit into the ac equivalent network of  Fig.   5.52   .    


background image

287

 In other words, the addition of a load resistor  

L

   to  the  configuration  of   Fig.    5.54a     will 

always have the effect of reducing the gain below the no-load level. 

 Furthermore: 

  The gain obtained with a source resistance in place will always be less than that 
obtained under loaded or unloaded conditions due to the drop in applied voltage across 
the source resistance.  

 In total, therefore, the highest gain is obtained under no-load conditions and the lowest 

gain with a source impedance and load in place. That is: 

  For the same configuration    A

v

NL

A

v

L

A

v

s

.     

 It will also be interesting to verify that: 

  For a particular design, the larger the level of R 

L

 , the greater is the level of ac gain.  

 In other words, the larger the load resistance, the closer it is to an open-circuit approxi-

mation that would result in the higher no-load gain. 

 In  addition: 

  For a particular amplifier, the smaller the internal resistance of the signal source, the 
greater is the overall gain.  

 In other words, the closer the source resistance is to a short-circuit approximation, the 

greater is the gain because the effect of  

s

   will essentially be eliminated. 

 For any network, such as those shown in  Fig.   5.54    that have coupling capacitors, the 
source and load resistance do not affect the dc biasing levels. 

 The conclusions listed above are all quite important in the amplifier design process. 

When one purchases a packaged amplifier, the listed gain and all the other parameters are 
for the  unloaded situation.  The gain that results due to the application of a load or source 
resistance can have a dramatic effect on all the amplifier parameters, as will be demon-
strated in the examples to follow. 

 In general, there are two directions one can take to analyze networks with an applied 

load and/or source resistance. One approach is to simply insert the equivalent circuit, as 
was demonstrated in  Section   5.11   , and use methods of analysis to determine the quantities 
of interest. The second is to define a two-port equivalent model and use the parameters 
determined for the no-load situation. The analysis to follow in this section will use the first 
approach, leaving the second method for  Section    5.14   . 

 For the fixed-bias transistor amplifier of  Fig.   5.54c   , substituting the  

e

   equivalent circuit 

for the transistor and removing the dc parameters results in the configuration of  Fig.    5.55   . 

 

R

 

R

 

R

 

V

 

+

V

 

+

V

CC

(b)

R

 

R

 

V

 

+

V

 

+

V

CC

A

v

NL

 

=

V

o

V

i

A

v

s

 

=

V

o

V

s

A

v

L

 

=

V

o

V

i

(a)

R

 

R

 

V

 

+

R

 

V

CC

+

V

s

R

 

(c)

 FIG. 5.54 

         Amplifier configurations: (a) unloaded; (b) loaded; (c) loaded with a source resistance.    


background image

BJT AC ANALYSIS

288

 

 It is particularly interesting that  Fig.   5.55    is exactly the same in appearance as  Fig.   5.22    

except that now there is a load resistance in parallel with  

C

   and a source resistance has 

been introduced in series with a source  

s

  . 

 The parallel combination of 

    

 R

L

r

o

7

R

C

7

R

L

⬵ R

C

7

R

L

   

 and     

 V

o

= -bI

b

R

L

= -bI

b

(R

C

7

R

L

)   

 with     

 I

b

=

V

i

br

e

   

 gives     

 V

o

= -ba

V

i

br

e

b(R

C

7

R

L

)

 

 

 

 so  that     

A

v

L

=

V

o

V

i

= -

R

C

R

L

r

e

  

(5.73)    

 The only difference in the gain equation using  

i

   as the input voltage is the fact that  

C

   

of Eq. (5.10) has been replaced by the parallel combination of  

C

   and  

L

  . This makes good 

sense because the output voltage of  Fig.   5.55    is now across the parallel combination of the 
two resistors. 

 The input impedance is 

    

Z

i

R

B

7

br

e

  

(5.74)    

 as before, and the output impedance is 

    

Z

o

R

C

7

r

o

  

(5.75)    

 as  before. 

 If the overall gain from signal source  

s

   to output voltage  

o

   is desired, it is only neces-

sary to apply the voltage-divider rule as follows: 

    

 V

i

=

Z

i

V

s

Z

i

R

s

   

 and     

 

V

i

V

s

=

Z

i

Z

i

R

s

   

 or     

 A

v

S

=

V

o

V

s

=

V

o

V

i

#

V

i

V

s

A

v

L

 

Z

i

Z

i

R

s

   

 so  that     

A

v

S

=

Z

i

Z

i

R

s

A

v

L

  

(5.76)    

 Because the factor    Z

i

>(Z

i

R

s

)    must always be less than one, Eq. (5.76) clearly supports 

the fact that the signal gain    A

v

S

    is always less than the loaded gain    A

v

L

.    

+

V

i

+

V

o

R

 

R

B 

Z

o

Z

i

r

o

R

C

R

L

I

b

β

r

e

β

R

L

 

= r

o    

R

C    

R

≅ R

C   

R

L

I

b

+

V

s 

 FIG. 5.55 

          The  ac  equivalent  network  for  the  network  of   Fig.    5.54c   .    


background image

289

EFFECT OF R

L

 AND R

S

  

EXAMPLE 5.11 

  Using the parameter values for the fixed-bias configuration of  Example   5.1    

with an applied load of    4.7 k

    and a source resistance of    0.3 k,     determine  the  following 

and compare to the no-load values: 

    a.       A

v

L

.     

   b.       A

v

s

.     

   c.     

i

  .  

   d.     

o

  .   

  

Solution:  

 

 

     a.    Eq.  (5.73):     A

v

L

= -

R

C

R

L

r

e

= -

3 k

7 4.7 k

10.71 

= -

1.831 k

10.71 

= ⴚ170.98    

     which is significantly less than the no-load gain of 

-280.11.  

    b.    Eq.  (5.76):     A

v

s

=

Z

i

Z

i

R

s

A

v

L

    

     With     Z

i

= 1.07 k     from   Example    5.1   ,  we  have 

    

A

v

s

=

1.07 k

1.07 k

 + 0.3 k

 (

-170.98) = ⴚ133.54   

     which again is significantly less than    A

v

NL

    or    A

v

L

.     

   c.       Z

i

1.07 k⍀    as obtained for the no-load situation.  

   d.       Z

o

R

C

3 k⍀    as obtained for the no-load situation. 

     The example clearly demonstrates that    A

v

NL

A

v

L

A

v

s

.        

 For the voltage-divider configuration of  Fig.   5.56    with an applied load and series source 

resistor the ac equivalent network is as shown in  Fig.    5.57   . 

  

C

1

 V

CC

R

s

R

1

R

2

R

C

R

L

R

E

V

i

C

E

C

2

+

V

o

+

Z

 

I

 

Z

 

V

s

+

 FIG. 5.56 

          Voltage-divider  bias  configuration  with  R 

s

  and R 

L

 .    

I

b

β  

I

b 

I

o

R'

I

i

R

s

b

c

e

e

R

 

+

V

 

+

V

 

+

V

s 

r

 

r

e

β

R

2

R

1

Z

i

Z

 

R

L

 FIG. 5.57 

          Substituting  the  r 

e

  equivalent circuit into the ac equivalent network of  Fig.   5.56   .    


background image

BJT AC ANALYSIS

290

 

 First note the strong similarities with  Fig.   5.55   , with the only difference being the par-

allel connection of  R  

1

  and  R  

2

  instead of just  

B

  . Everything else is exactly the same. The 

following equations result for the important parameters of the configuration: 

    

A

v

L

=

V

o

V

i

= -

R

C

R

L

r

e

  

(5.77)    

    

Z

i

R

1

7

R

2

7

br

e

  

(5.78)    

    

Z

o

R

C

7

r

o

  

(5.79)    

 For the emitter-follower configuration of  Fig.   5.58    the small-signal ac equivalent net-

work is as shown in  Fig.   5.59   . The only difference between  Fig.   5.59    and the unloaded 
configuration of  Fig.   5.37    is the parallel combination of  

E

   and  

L

   and the addition of the 

source resistor  

s

  . The equations for the quantities of interest can therefore be determined 

by simply replacing  

E

   by     R

E

7

R

L

     wherever   

E

   appears. If  

E

   does not appear in an equation, 

the load resistor  

L

   does not affect that parameter. That is, 

    

A

v

L

=

V

o

V

i

=

R

E

R

L

R

E

R

L

r

e

  

(5.80)    

R

 

B

C

C

1

C

2

R

 

R

 

V

CC

V

 

V

o 

+

I

i

I

o

Z

o

+

V

s

R

 

Z

 

 FIG. 5.58 

          Emitter-follower  configuration  with  R 

s

  and R 

L

 .    

+

V

s

R

 

R

E

R

L

R

B 

c

e

r

e

β

b

I

i

Z

i

I

o

Z

o

I

b 

I

b

β 

V

 

+

V

o

+

 FIG. 5.59 

          Substituting  the  r 

e

  equivalent circuit into the ac equivalent network of  Fig.   5.58   .    


background image

291

DETERMINING THE 

CURRENT GAIN

    

Z

i

R

B

7

Z

b

  

(5.81)

    

    

Z

b

⬵ b(R

E

7

R

L

)  

(5.82)    

    

Z

o

⬵ r

e

  

(5.83)

    

 The effect of a load resistor and a source impedance on the remaining BJT configura-

tions  will  not  be  examined  in  detail  here,  although   Table    5.1     in   Section    5.14     will  review 
the results for each configuration. 

  

 

   5.13 

 DETERMINING  THE  CURRENT  GAIN 

 ● 

 You may have noticed in the previous sections that the current gain was not determined for 
each configuration. Earlier editions of this text did have the details of finding that gain, but 
in reality the voltage gain is usually the gain of most importance. The absence of the deri-
vations should not cause concern because: 

  For each transistor configuration, the current gain can be determined directly from the 
voltage gain, the defined load, and the input impedance.  

 The derivation of the equation linking the voltage and current gains can be derived using 

the two-port configuration of  Fig.    5.60   . 

 

R

 

V

 

Z

 

I

 

I

o 

+

+

V

 

Z

 

System

 FIG. 5.60 

        Determining  the  current  gain  using  the  voltage  gain.    

 The current gain is defined by 

    

A

i

=

I

o

I

i

  

(5.84)    

 Applying Ohm’s law to the input and output circuits results in 

    

I

i

=

V

i

Z

i

  and  I

o

= -

V

o

R

L

   

 The minus sign associated with the output equation is simply there to indicate that the polar-

ity of the output voltage is determined by an output current having the opposite direction. By 
definition, the input and output currents have a direction entering the two-port configuration. 

 Substituting into Eq. (5.84) then results in 

    

A

i

L

=

I

o

I

i

=

-

V

o

R

L

V

i

Z

i

= -

V

o

V

i

#

Z

i

R

L

   

 and the following important equation: 

    

A

i

L

= -A

v

L

Z

i

R

L

  

(5.85)    

 The value of  

L

   is defined by the location of  

o

   and  

o

  . 


background image

BJT AC ANALYSIS

292

 

 To demonstrate the validity of Eq. (5.82), consider the voltage-divider bias configura-

tion  of   Fig.    5.28   . 

 Using  the  results  of   Example    5.2   ,  we  find 

    

I

i

=

V

i

Z

i

=

V

i

1.35 k

 and I

o

= -

V

o

R

L

= -

V

o

6.8 k

   

 so  that     

 A

i

L

=

I

o

I

i

=

a-

V

o

6.8 k

b

V

i

1.35 k

= - a

V

o

V

i

b a

1.35 k

6.8 k

b

 

 

= -(-368.76)a

1.35 k

6.8 k

b = 73.2

   

 Using  Eq.  5.82:  

   A

i

L

= -A

v

L

Z

i

R

L

= -(-368.76)a

1.35 k

6.8 k

b = 73.2   

 which has the same format as the resulting equation above and the same result. 

 The solution to the current gain in terms of the network parameters will be more com-

plicated for some configurations if a solution is desired in terms of the network parameters. 
However, if a numerical solution is all that is desired, it is simply a matter of substituting 
the value of the three parameters from an analysis of the voltage gain. 

 As a second example, consider the common-base bias configuration of  Section   5.9   . In 

this case the voltage gain is 

    

A

v

L

R

C

r

e

   

 and the input impedance is 

    

Z

i

⬵ R

E

7

r

e

⬵ r

e

   

 with   

L

   defined as  

C

   due to the location of  

o

  . 

 The result is the following: 

    

A

i

L

= -A

v

L

Z

i

R

L

= a-

R

C

r

e

b a

r

e

R

C

b ⬵ -1   

 which agrees with the solution of that section because    I

c

⬵ I

e

.    Note, in this case, that the 

output current has the opposite direction to that appearing in the networks of that section 
due to the minus sign.  

   5.14 

 SUMMARY  TABLES 

 ● 

 The last few sections have included a number of derivations for unloaded and loaded BJT 
configurations. The material is so extensive that it seemed appropriate to review most of 
the conclusions for the various configurations in summary tables for quick comparisons. 
Although the equations using the hybrid parameters have not been discussed in detail at 
this point, they are included to make the tables complete. The use of hybrid parameters 
will be considered in a later section of this chapter. In each case the waveforms included 
demonstrate the phase relationship between input and output voltages. They also reveal the 
relative magnitude of the voltages at the input and output terminals. 

  Table   5.1    is for the unloaded situation, whereas  Table   5.2    includes the effect of  

s

   and  

L

  .                   

 

 

 

         

   5.15 

 TWO-PORT  SYSTEMS  APPROACH 

 ● 

 In the design process, it is often necessary to work with the terminal characteristics of a 
device rather then the individual components of the system. In other words, the designer is 
handed a packaged product with a list of data regarding its characteristics but has no access 
to the internal construction. This section will relate the important parameters determined 
for a number of configurations in the previous sections to the important parameters of this 
packaged system. The result will be an understanding of how each parameter of the 


background image

R

C

R

F

V

CC

V

Z

o

+

I

o

I

o

V

o

+

I

i

Z

o

      

R

E

R

B

V

CC

I

o

V

o

+

Z

i

I

i

V

+

Z

o

R

B

R

C

R

E

V

CC

I

i

I

o

V

o

+

V

+

Z

i

Z

o

R

2  

R

1  

R

C

R

E

V

CC

C

E

Z

i

I

i

I

o

V

o

+

V

+

Z

o

 TABLE 5.1 

 Unloaded BJT Transistor Amplifiers 

 Configuration 

 Z 

i

  

 Z 

o

  

 A 

v

  

 A 

i

  

 Fixed-bias: 

    Medium (1 k

       

=   R

B

7

br

e

        

⬵   br

e

        (R

B

Ú 10br

e

)    

    Medium (2 k

)

        

=   R

C

7

r

o

        

⬵   R

C

        (r

o

Ú 10R

C

)    

    High (

-200)

        

=   -

(R

C

7

r

o

)

r

e

        

⬵   -

R

C

r

e

        (r

o

Ú 10R

C

)    

    High (100)

        

=   

bR

B

r

o

(r

o

R

C

)(R

B

+ br

e

)

        

⬵   b

        (r

o

Ú 10R

C

,

        R

B

Ú 10br

e

)    

 Voltage-divider 
bias: 

    Medium (1 k

)

        

=   R

1

7

R

2

7

br

e

    

    Medium (2 k

)

        

=   R

C

7

r

o

        

⬵   R

C

        (r

o

Ú 10R

C

)    

    High (

-200)

        

=   -

R

C

7

r

o

r

e

        

⬵   -

R

C

r

e

        (r

o

Ú 10R

C

)    

    High (50)

        

=   

b(R

1

7

R

2

)r

o

(r

o

R

C

)(R

1

7

R

2

+ br

e

)

        

⬵   

b(R

1

7

R

2

)

R

1

7

R

2

+ br

e

        (r

o

Ú 10R

C

)    

 Unbypassed 
emitter bias: 

    High (100 k

)

        

=   R

B

7

Z

b

        Z

b

⬵ b(r

e

R

E

)

        

⬵   R

B

7

bR

E

        (R

E

r

e

)    

    Medium (2 k

)

        

=   R

C

        (any level of r

o

)    

    Low (

-5)

        

=   -

R

C

r

e

R

E

        

⬵  -

R

C

R

E

    

    (R

E

r

e

)    

    High (50)    

    

⬵  -

bR

B

R

B

Z

b

    

 Emitter-
follower: 

    High (100 k

)

        

=   R

B

7

Z

b

        Z

b 

⬵  b(r

e

R

E

)

        

⬵   R

B

7

bR

E

        (R

E

r

e

)    

    Low (20 

)    

    

=  R

E

7

r

e

    

    

⬵  r

e

    

    (R

E

r

e

)    

    Low (

⬵1)    

    

=  

R

E

R

E

r

e

    

    

⬵  1    

    High (

-50)    

    

⬵  -

bR

B

R

B

Z

b

    

 Common-base: 

    Low (20 

)

        

=   R

E

7

r

e

        

⬵  r

e

        (R

E

r

e

)    

    Medium (2 k

)    

    

=  R

C

    

    High (200)    

    

⬵  

R

C

r

e

    

    Low (

-1)    

    

⬵  -1    

 Collector 
feedback: 

    Medium (1 k

)

        

=  

r

e

1

b

+

R

C

R

F

        (r

o

Ú 10R

C

)    

    Medium (2 k

)    

    

⬵  R

C

7

R

F

    

    (r

o

Ú 10R

C

)    

    High (

-200)    

    

⬵  -

R

C

r

e

    

    (r

o

Ú 10R

C

)    

    (R

F

R

C

)    

    High (50)    

    

=  

bR

F

R

F

+ bR

C

    

    

⬵  

R

F

R

C

    

R

B

R

C

V

CC

V

Z

i

V

o

+

+

I

i

I

o

Z

o

R

E

R

C

V

EE

V

CC

I

o

V

o

+

Z

i

V

+

I

i

Z

o

293


background image

 TABLE 5.2 

 BJT Transistor Amplifiers Including the Effect of R 

s

  and R 

L

  

 Configuration 

    A

v

L

ⴝ V

o

>V

i

    

 Z 

i

  

 Z 

o

  

    

-(R

L

R

C

)

r

e

    

    R

B

7

br

e

    

  

C

   

 Including   

o

  : 

    

-

(R

L

7

R

C

7

r

o

)

r

e

    

    R

B

7

br

e

    

    R

C

7

r

o

    

    

-(R

L

7

R

C

)

r

e

    

    R

1

7

R

2

7

br

e

    

  

C

   

 Including   

o

  : 

    

-(R

L

7

R

C

7

r

o

)

r

e

    

    R

1

7

R

2

7

br

e

    

    R

C

7

r

o

    

    

⬵ 1    

    R

E

R

L

7

R

E

    

    R

1

7

R

2

7

b(r

e

R

E

)    

    R

s

R

s

7

R

1

7

R

2

    

    R

E

a

R

s

b

r

e

b    

 Including   

o

  : 

    

⬵ 1    

    R

1

7

R

2

7

b(r

e

R

E

)    

    R

E

a

R

s

b

r

e

b    

    

-(R

L

7

R

C

)

r

e

    

    R

E

7

r

e

    

  

C

   

 Including   

o

  : 

    

-(R

L

7

R

C

7

r

o

)

r

e

    

    R

E

7

r

e

    

    R

C

7

r

o

    

    

-(R

L

7

R

C

)

R

E

    

    R

1

7

R

2

7

b(r

e

R

E

)    

  

C

   

 Including   

o

  : 

    

-(R

L

7

R

C

)

R

E

    

    R

1

7

R

2

7

b(r

e

R

e

)    

    

⬵ R

C

    

      

V

o

V

CC

R

C

Z

o

R

s

R

1

V

i

R

E

R

2

R

L

V

s

+

Z

i

      

      

      

      

294


background image

packaged system relates to the actual amplifier or network. The system of  Fig.   5.61    is 
called a two-port system because there are two sets of terminals—one at the input and the 
other at the output. At this point it is particularly important to realize that 

 

  the data surrounding a packaged system is the no-load data.  

 This should be fairly obvious because the load has not been applied, nor does it come with 
the load attached to the package. 

 Configuration 

    A

v

L

ⴝ V

o

>V

i

    

 Z 

i

  

 Z 

o

  

    

-(R

L

7

R

C

)

R

E

1

    

    R

B

7

b(r

e

R

E

1

)    

  

C

   

 Including   

o

  : 

    

-(R

L

7

R

C

)

R

E

t

    

    R

B

7

b(r

e

R

E

)    

    

⬵ R

C

    

    

-(R

L

7

R

C

)

r

e

    

    

br

e

R

F

兩 A

v

    

  

C

   

 Including   

o

  : 

    

-(R

L

7

R

C

7

r

o

)

r

e

    

    

br

e

R

F

0

A

v

0

    

    R

C

7

R

F

7

r

o

    

    

-(R

L

7

R

C

)

R

E

    

    

bR

E

R

F

0

A

v

0

    

    

⬵ R

C

7

R

F

    

 Including   

o

  : 

    

-(R

L

7

R

C

)

R

E

    

    

⬵ bR

E

R

F

0

A

v

0

    

    

⬵ R

C

7

R

F

    

V

o

V

CC

R

C

Z

o

C

E

R

s

V

i

R

B

R

E2

R

L

V

s

+

Z

i

R

E1

      

V

o

V

CC

V

s

R

C

Z

o

R

L

R

s

V

i

+

Z

i

R

F

      

V

o

V

CC

V

s

R

C

Z

o

R

s

V

i

+

Z

i

R

F

R

L

R

E

R

L

      

TABLE 5.2 (Continued)

BJT Transistor Amplifiers Including the Effect of R

s

 and R

L

+

V

 

Z

 

I

 

I

 

+

– 

Z

 

V

 

Thévenin

A

v

NL

 FIG. 5.61 

        Two-port  system.    

295


background image

BJT AC ANALYSIS

296

 

 For the two-port system of  Fig.   5.61    the polarity of the voltages and the direction of 

the currents are as defined. If the currents have a different direction or the voltages have 
a different polarity from that appearing in  Fig.   5.61   , a negative sign must be applied. 
Note again the use of the label    A

v

NL

    to indicate that the provided voltage gain will be the 

no-load value. 

 For amplifiers the parameters of importance have been sketched within the boundaries 

of the two-port system as shown in  Fig.   5.62   . The input and output resistance of a packaged 
amplifier are normally provided along with the no-load gain. They can then be inserted as 
shown in  Fig.   5.62    to represent the seated package. 

 

A

v

NL

V

i

 FIG. 5.62 

         Substituting  the  internal  elements  for  the  two-port  system  of  Fig.  5.61.    

 For the no-load situation the output voltage is 

 

   V

o

A

v

NL

V

i

  

(5.86)    

 due to the fact that  I 

   0A , resulting in    I

o

R

o

= 0V.    

 The output resistance is defined by  

i

  

   0V . Under such conditions the quantity    A

v

NL

V

i

    

is zero volts also and can be replaced by a short-circuit equivalent. The result is 

 

   Z

o

R

o

  

(5.87)    

 Finally, the input impedance  

i

   simply relates the applied voltage to the resulting input 

current and 

 

   Z

i

R

i

  

(5.88)

    

 For the no-load situation, the current gain is undefined because the load current is zero. 

There is, however, a no-load voltage gain equal to    A

v

NL

.    

 The effect of applying a load to a two-port system will result in the configuration of 

 Fig.    5.63   . Ideally, all the parameters of the model are unaffected by changing loads and 
levels of source resistance. However, for some transistor configurations the applied load 
can affect the input resistance, whereas for others the output resistance can be affected by 
the source resistance. In all cases, however, by simple definition, the no-load gain is unaf-
fected by the application of any load. In any case, once    A

v

NL

,      

i

  , and  

o

   are defined for a 

particular configuration, the equations about to be derived can be employed. 

 

A

v

NL

Vi

 FIG. 5.63 

        Applying  a  load  to  the  two-port  system  of   Fig.    5.62   .    


background image

297

TWO-PORT SYSTEMS 

APPROACH

 Applying the voltage-divider rule to the output circuit results in 

 

   V

o

=

R

L

A

v

NL

V

i

R

L

R

o

   

 and  

   A

v

L

=

V

o

V

i

=

R

L

R

L

R

o

A

v

NL

  

(5.89)

    

 Because the ratio    R

L

>(R

L

R

o

)    is always less than 1, we have further evidence that the 

loaded voltage gain of an amplifier is always less than the no-load level. 

 The current gain is then determined by 

 

   A

i

L

=

I

o

I

i

=

-V

o

>R

L

V

i

>Z

i

= -

V

o

V

i

 

Z

i

R

L

   

 and  

   A

i

L

= -A

v

L

Z

i

R

L

  

(5.90)

    

 as obtained earlier. In general, therefore, the current gain can be obtained from the voltage 
gain and impedance parameters  

i

   and  

L

  . The next example will demonstrate the useful-

ness and validity of Eqs. (5.89) and (5.90). 

 Our attention will now turn to the input side of the two-port system and the effect of an 

internal source resistance on the gain of an amplifier. In  Fig.   5.64   , a source with an internal 
resistance has been applied to the basic two-port system. The definitions of  

i

   and     A

v

NL

     are 

such that: 

  The  parameters  Z 

i

  and    A

v

NL

    of a two-port system are unaffected by the internal resis-

tance of the applied source.  

 

V

s

+

Z

i

Z

o

V

i

+

V

o

+

I

i

I

o

I

s

A

NL

Vi

 FIG. 5.64 

          Including  the  effects  of  the  source  resistance  R 

s

 .    

 However: 

  The output impedance may be affected by the magnitude of R 

s

 .  

 The fraction of the applied signal reaching the input terminals of the amplifier of  Fig.   5.64    

is determined by the voltage-divider rule. That is, 

 

   V

i

=

R

i

V

s

R

i

R

s

  

(5.91)    

 Equation (5.91) clearly shows that the larger the magnitude of  

s

  , the lower is the voltage 

at the input terminals of the amplifier. In general, therefore, as mentioned earlier, for a 
particular amplifier, the larger the internal resistance of a signal source, the lower is the 
overall gain of the system. 

 For  the  two-port  system  of   Fig.    5.64   , 

 

    V

o

A

v

NL

V

i

   

 and  

     V

i

=

R

i

V

s

R

i

R

s

   


background image

BJT AC ANALYSIS

298

 

 so  that  

     V

o

A

v

NL

R

i

R

i

R

s

V

s

   

 and  

    A

v

s

=

V

o

V

s

=

R

i

R

i

R

s

A

v

NL

  

(5.92)    

 The effects of  

s

   and  

L

   have now been demonstrated on an individual basis. The next 

natural question is how the presence of both factors in the same network will affect the 
total gain. In  Fig.   5.65   , a source with an internal resistance  

s

   and a load  

L

   have been 

applied to a two-port system for which the parameters  

i

  ,    A

v

NL

,     and   

o

   have been specified. 

For the moment, let us assume that  

i

   and  

o

   are unaffected by  

L

   and  

s

  ,  respectively. 

 

I

s

+

V

s

I

 

R

 

V

 

+

Z

o

I

o

 FIG. 5.65 

        Considering  the  effects  of  R 

s

  and R 

L

  on the gain of an amplifier.    

 At the input side we find 

 

 Eq.  (5.91): 

   V

i

=

R

i

V

s

R

i

R

s

    

 or  

   

V

i

V

s

=

R

i

R

i

R

s

  

(5.93)    

 and at the output side, 

 

   V

o

=

R

L

R

L

R

o

 A

v

NL

V

i

   

 or  

   A

v

L

=

V

o

V

i

=

R

L

A

v

NL

R

L

R

o

=

R

L

R

L

R

o

A

v

NL

  

(5.94)    

 For the total gain    A

v

s

V

o

>V

s

,    the following mathematical steps can be performed: 

 

   A

v

s

=

V

o

V

s

=

V

o

V

i

#

V

i

V

s

  

(5.95)    

 and substituting Eqs. (5.93) and (5.94) results in 

 

   A

v

s

=

V

o

V

s

=

R

i

R

i

R

s

#

R

L

R

L

R

o

A

v

NL

  

(5.96)    

 Because     I

i

V

i

>R

i

,     as  before, 

 

   A

i

L

= -A

v

L

R

i

R

L

  

(5.97)    

 or,  using     I

s

V

s

>(R

s

R

i

),    

 

   A

i

s

= -A

v

s

R

s

R

i

R

L

  

(5.98)    


background image

299

TWO-PORT SYSTEMS 

APPROACH

 However,     I

i

I

s

,    so Eqs. (5.97) and (5.98) generate the same result. Equation (5.96) 

clearly reveals that both the source and the load resistance will reduce the overall gain of 
the system. 

 The two reduction factors of Eq. (5.96) form a product that has to be carefully consid-

ered in any design procedure. It is not sufficient to ensure that  

s

   is relatively small if the 

effect of the magnitude of  

L

   is ignored. For instance, in Eq. (5.96), if the first factor is 0.9 

and the second factor is 0.2, the product of the two results in an overall reduction factor 
equal to (0.9)(0.2)

  0.18, which is close to the lower factor. The effect of the excellent 

0.9 level was completely wiped out by the significantly lower second multiplier. If both 
were 0.9-level factors, the net result would be (0.9)(0.9)

  0.81, which is still quite high. 

Even if the first were 0.9 and the second 0.7, the net result of 0.63 would still be respect-
able. In general, therefore, for good overall gain the effects of  

s

   and  

L

   must be evaluated 

individually and as a product. 

  

EXAMPLE 5.12 

  Determine     A

v

L

     and     A

v

s

     for  the  network  of   Example    5.11     and  compare 

solutions.   Example    5.1     showed  that     A

v

NL

= -280,        Z

i

= 1.07 k,     and     Z

o

= 3 k.     In 

 Example    5.11   ,     R

L

= 4.7 k     and     R

s

= 0.3 k.    

  

Solution:  

 

 

     a.    Eq.  (5.89):     A

v

L

=

R

L

R

L

R

o

A

v

NL

    

 

    

=

4.7 k

4.7 k

 + 3 k

 (

-280.11)

 

 

= ⴚ170.98

   

     as  in   Example    5.11   .  

    b.    Eq.  (5.96):     A

v

s

=

R

i

R

i

R

s

#

R

L

R

L

R

o

A

v

NL

    

 

    

=

1.07 k

1.07 k

 + 0.3 k

#

4.7 k

4.7 k

 + 3 k

 (

-280.11)

 

 

= (0.781)(0.610)(-280.11)

 

 

= ⴚ133.45   

     as  in   Example    5.11   .     

  

EXAMPLE 5.13 

  Given  the  packaged  (no-entry-possible)  amplifier  of   Fig.    5.66   : 

     a.    Determine  the  gain     A

v

L

    and compare it to the no-load value with    R

L

= 1.2 k.     

    b.    Repeat  part  (a)  with     R

L

= 5.6 k     and  compare  solutions.  

    c.    Determine     A

v

s

     with     R

L

= 1.2 k.     

    d.    Find  the  current  gain     A

i

=

I

o

I

i

=

I

o

I

s

     with     R

L

= 5.6 k.      

 

R

 

R

s

I

 

V

 

V

i 

+

+

+

V

s

I

s

I

o

= 2 kΩ

Z

o

= 4 kΩ

Z

i

= –480

A

v

NL

0.2 k

Ω

 FIG. 5.66 

        Amplifier  for   Example    5.13   .    


background image

BJT AC ANALYSIS

300

 

  

Solution:  

 

 

     a.    Eq.  (5.89):     A

v

L

=

R

L

R

L

R

o

 A

v

NL

    

 

    

=

1.2 k

1.2 k

⍀ + 2 k⍀

 (

-480) = (0.375)(-480)

 

 

= ⴚ180   

     which is a dramatic drop from the no-load value.  

    b.    Eq.  (5.89):     A

v

L

=

R

L

R

L

R

o

 A

v

NL

    

 

    

=

5.6 k

5.6 k

⍀ + 2 k⍀

 (

-480) = (0.737)(-480)

 

 

= ⴚ353.76

   

     which clearly reveals that the larger the load resistor, the better is the gain.  

    c.    Eq.  (5.96):     A

v

s

=

R

i

R

i

R

s

#

R

L

R

L

R

o

 A

v

NL

    

 

    

=

4 k

4 k

⍀ + 0.2 k⍀

#

1.2 k

1.2 k

⍀ + 2 k⍀

 (

-480)

 

 

= (0.952)(0.375)(-480)

 

 

= ⴚ171.36   

     which is fairly close to the loaded gain  

v

   because the input impedance is considerably 

more than the source resistance. In other words, the source resistance is relatively 
small compared to the input impedance of the amplifier.  

   d.        A

i

L

=

I

o

I

i

=

I

o

I

s

= -A

v

L

Z

i

R

L

    

         

= -(-353.76)a

4 k

5.6 k

b = -(-353.76)(0.714)   

         

252.6       

 It is important to realize that when using the two-port equations in some configurations 

the input impedance is sensitive to the applied load (such as the emitter-follower and collec-
tor feedback) and in some the output impedance is sensitive to the applied source resistance 
(such as the emitter-follower). In such cases the no-load parameters for  

i

   and  

o

   have to 

first be calculated before substituting into the two-port equations. For most packaged sys-
tems such as op-amps this sensitivity of the input and output parameters to the applied load 
or source resistance is minimized to eliminate the need to be concerned about changes from 
the no-load levels when using the two-port equations.  

   5.16 

 CASCADED  SYSTEMS 

 ● 

 The two-port systems approach is particularly useful for cascaded systems such as that 
appearing  in   Fig.    5.67   ,  where     A

v

1

A

v

2

A

v

3

,    and so on, are the voltage gains of each stage 

 under loaded conditions.  That is,    A

v

1

    is determined with the  input impedance to     A

v

2

     acting 

as the load on     A

v

1

.     For     A

v

2

,       A

v

1

    will determine the signal strength and source impedance at 

the input to    A

v

2

.    The total gain of the system is then determined by the product of the indi-

vidual gains as follows: 

 

   A

v

T

A

v

1

#

A

v

2

#

A

v

3

 . . . .  

(5.99)

    

 and the total current gain is given by 

 

   A

i

T

= -A

v

T

Z

i

1

R

L

  

(5.100)    


background image

301

CASCADED SYSTEMS

 No matter how perfect the system design, the application of a succeeding stage or load 

to a two-port system will affect the voltage gain. Therefore, there is no possibility of a 
situation  where     A

v

1

A

v

2

,    and so on, of  Fig.   5.67    are simply the no-load values. The no-load 

parameters can be used to determine the loaded gains of each stage, but Eq. (5.99) requires 
the loaded values. The load on stage 1 is    Z

i

2

,     on  stage  2     Z

i

3

,     on  stage  3     Z

i

n

,     and  so  on. 

 

V

 

Z

i

 = Z

i

1

=

V

o

1

V

i

2

+

R

L

Z

i

2

Z

i

3

Z

o

1

Z

o

2

Z

o

3

=

V

o

2

V

i

3

V

 

+

A

v

1

Z

in

Z

on

 = 

Z

o

A

v

2

A

v

3

A

v

n

 FIG. 5.67 

        Cascaded  system.    

  

EXAMPLE 5.14 

  The two-stage system of  Fig.   5.68    employs a transistor emitter-follower 

configuration prior to a common-base configuration to ensure that the maximum percentage 
of the applied signal appears at the input terminals of the common-base amplifier. In  Fig. 
  5.68   , the no-load values are provided for each system, with the exception of  

i

   and  

o

   for the 

emitter-follower, which are the loaded values. For the configuration of  Fig.   5.68   , determine: 

     a.    The  loaded  gain  for  each  stage.  
   b.    The total gain for the system,  

v

   and     A

v

s

.     

   c.    The total current gain for the system.  
   d.    The total gain for the system if the emitter-follower configuration were removed.   

 

  

Solution:  

 

 

    a.    For the emitter-follower configuration, the loaded gain is (by Eq. (5.94)) 

 

    V

o

1

=

Z

i

2

Z

i

2

Z

o

1

 A

v

NL

V

i

1

=

26 

26 

 + 12 

 (1) V

i

1

= 0.684 V

i

1

   

 and  

    A

V

i

=

V

o

1

V

i

1

0.684

   

     For the common-base configuration, 

 

    V

o

2

=

R

L

R

L

R

o

2

A

v

NL

 V

i

2

=

8.2 k

8.2 k

 + 5.1 k

 (240) V

i

2

= 147.97 V

i

2

   

 and  

    A

v

2

=

V

o

2

V

i

2

147.97

    

    b.    Eq.  (5.99):     A

v

T

A

v

1

A

v

2

    

        

 

= (0.684)(147.97)

     

 

101.20   

R

L

V

 

+

 FIG. 5.68 

         Example    5.14   .    


background image

BJT AC ANALYSIS

302

 

 Eq.  (5.91):       A

v

s

=

Z

i

1

Z

i

1

R

s

A

v

T

=

(10 k

⍀)(101.20)

10 k

⍀ + 1 k⍀

     

 

92

    

    c.    Eq.  (5.100):      A

i

T

= -A

v

T

Z

i

1

R

L

= -(101.20)a

10 k

8.2 k

b    

     

     

= ⴚ123.41

    

    d.    Eq.  (5.91):       V

i

=

Z

i

CB

Z

i

CB

R

s

V

s

=

26 

26 

⍀ + 1 k⍀

V

s

= 0.025 V

s

    

 and 

     

V

i

V

s

= 0.025  with  

V

o

V

i

= 147.97  from  above          

 and 

    A

v

s

=

V

o

V

s

=

V

i

V

s

#

V

o

V

i

= (0.025)(147.97) = 3.7

      

 In total, therefore, the gain is about 25 times greater with the emitter-follower configuration 
to draw the signal to the amplifier stages. Note, however, that it is also important that the 
output impedance of the first stage is relatively close to the input impedance of the second 
stage, otherwise the signal would have been “lost” again by the voltage-divider action.   

  RC-Coupled BJT Amplifiers 

 One popular connection of amplifier stages is the  RC -coupled  variety  shown  in   Fig.    5.69     in 
the next example. The name is derived from the capacitive coupling capacitor  

c

   and the 

fact that the load on the first stage is an  RC  combination. The coupling capacitor isolates 
the two stages from a dc viewpoint but acts as a short-circuit equivalent for the ac response. 
The input impedance of the second stage acts as a load on the first stage, permitting the 
same approach to the analysis as described in the last two sections. 

  

EXAMPLE 5.15 

 

    a.    Calculate the no-load voltage gain and output voltage of the  RC -coupled  transistor 

amplifiers of  Fig.    5.69   .  

   b.    Calculate the overall gain and output voltage if a 4.7 k

⍀ load is applied to the second 

stage, and compare to the results of part (a).  

   c.    Calculate the input impedance of the first stage and the output impedance of the second 

stage.   

 

+20 V

2.2 k

Ω

20   F

μ

4.7 k

Ω

15 k

Ω

Q

1

C

C

Q

2

20   F

μ

2.2 k

Ω

15 k

Ω

4.7 k

Ω

1 k

Ω

10 

μF

10 

μF

10 

μF

V

i

 

= 25   V

μ

V

o

+

+

1 k

Ω

β = 200

β = 200

 FIG. 5.69 

        RC-coupled  BJT  amplifier  for   Example    5.15   .    

  

Solution:  

 

 

    a.    The dc bias analysis results in the following for each transistor: 

 

   V

B

= 4.8 V, V

E

= 4.1 V, V

C

= 11 V, I

E

= 4.1 mA   


background image

303

CASCADED SYSTEMS

     At the bias point, 

 

   r

e

=

26 mV

I

E

=

26 mV

4.1 mA

= 6.34 ⍀   

     The loading of the second stage is 

 

   Z

i

2

R

1

7

R

2

7

br

e

   

     which results in the following gain for the first stage: 

 

    A

v

1

= -

R

C

7 (R

1

R

2

7 br

e

)

r

e

 

 

= -

(2.2 k

⍀)

7 [15 k⍀ 7 4.7 k⍀ 7 (200)(6.34 ⍀)]

6.34 

 

 

= -

659.2 

6.34 

= -104

   

     For the unloaded second stage the gain is 

 

   A

v

2(NL)

= -

R

C

r

e

= -

2.2  k

6.34 

= -347   

     resulting in an overall gain of 

 

   A

v

T(NL)

A

v

1

A

v

2(NL)

= (-104)(-347) ⬵ 36.1 10

3

   

     The output voltage is then 

 

   V

o

A

v

T(NL)

V

i

= (36.1 * 10

3

)(25 

mV) ⬵ 902.5 mV    

   b.    The overall gain with the 10-k

⍀ load applied is 

 

   A

v

T

=

V

o

V

i

=

R

L

R

L

Z

o

A

v

T(NL)

=

4.7 k

4.7 k

⍀ + 2.2 k⍀

 (36.1

* 10

3

)

⬵ 24.6 10

3

   

     which is considerably less than the unloaded gain because  

L

   is relatively close to  

C

  . 

 

    V

o

A

v

T

V

i

 

 

= (24.6 * 10

3

)(25 

mV)

 

 

615 mV

    

   c.    The input impedance of the first stage is 

 

   Z

i

1

R

1

7

R

2

7

br

e

= 4.7 k⍀

7

15 k

7

(200)(6.34 

⍀) = 0.94 k⍀   

     whereas the output impedance for the second stage is 

 

   Z

o

2

R

C

2.2 k⍀        

  Cascode  Connection 

 The cascode configuration has one of two configurations. In each case the collector of the 
leading transistor is connected to the emitter of the following transistor. One possible 
arrangement appears in  Fig.   5.70   ; the second is shown in   Fig.    5.71    in the following example. 

V

i

V

o

 FIG. 5.70 

          Cascode  configuration.    


background image

BJT AC ANALYSIS

304

 

The arrangements provide a relatively high-input impedance with low voltage gain for the 
first stage to ensure the input Miller capacitance (to be discussed in  Section   9.9   ) is at a 
minimum, whereas the following CB stage provides an excellent high-frequency response. 

 

  

EXAMPLE 5.16 

  Calculate the no-load voltage gain for the cascode configuration of  Fig.   5.71   . 

 

V

CC  

=  18 V

V

2

= 5   F

μ

Q

2

V

1

Q

1

C

= 20 μF

R

E

1.1 k

Ω

6.8 k

Ω

R

B

1

10   F

μ

C

= 5   F

μ

V

i1

C

1

(    

=    200)

β

β

1

R

B

3

4.7 k

Ω

R

B

2

5.6 k

Ω

R

C

1.8 k

Ω

 FIG. 5.71 

        Practical  cascode  circuit  for   Example    5.16   .    

  

Solution:  

 

  The dc analysis results in 

 

   V

B

1

= 4.9 V, V

B

2

= 10.8 V, I

C

1

⬵ I

C

2

= 3.8 mA   

 because     I

E

1

⬵ I

E

2

    the dynamic resistance for each transistor is 

 

   r

e

=

26 mV

I

E

26 mV

3.8 mA

= 6.8    

 The loading on the transistor  Q  

1

  is the input impedance of the  Q  

2

  transistor in the CB 

configuration as shown by  

e

   in   Fig    5.72   . 

 The result is the replacement of  

C

   in the basic no-load equation for the gain of the CB 

configuration, with the input impedance of a CB configuration as follows: 

 

   A

v

1

= -

R

C

r

e

= -

r

e

r

e

= -1   

 with the voltage gain for the second stage (common base) of       

 

   A

v

2

=

R

C

r

e

=

1.8 k

6.8 

= 265   

 

V

i1

V

o1

V

o2

Q

1 

 

Q

2 

 

r

e

 FIG. 5.72 

          Defining  the  load  of  Q  

1

  .    


background image

 The overall no-load gain is 

 

   A

v

T

A

v

1

A

v

2

= (-1)(265) = ⴚ265   

 As expected, in  Example   5.16   , the CE stage provides a higher input impedance than can 

be expected from the CB stage. With a voltage gain of about 1 for the first stage, the 
Miller-effect input capacitance is kept quite low to support a good high-frequency response. 
A large voltage gain of 265 was provided by the CB stage to give the overall design a good 
input impedance level with desirable gain levels.     

   5.17 

 DARLINGTON  CONNECTION 

 ● 

 A very popular connection of two bipolar junction transistors for operation as one “super-
beta”  transistor  is  the  Darlington  connection  shown  in   Fig.    5.73   .  The  main  feature  of  the 
Darlington connection is that the composite transistor acts as a single unit with a current 
gain that is the product of the current gains of the individual transistors. If the connection 
is made using two separate transistors having current gains of    

b

1

     and     

b

2

,     the  Darlington 

connection provides a current gain of 

 

 

   

b

D

= b

1

b

2

  

(5.101)    

 FIG. 5.73 

        Darlington  combination.    

 The configuration was first introduced by Dr. Sidney Darlington in 1953. A short biog-

raphy appears as Fig 5.74. 

 

  Emitter-Follower  Configuration 

 A Darlington amplifier used in an emitter-follower configuration appears in  Fig.   5.75   . The 
primary impact of using the Darlington configuration is an input impedance much larger than 

      

  American  (Pittsburgh, PA; Exeter, NH) 
(1906–1997) 

  Department Head at Bell Laboratories  
Professor, Department of Electrical and 
Computer Engineering, University of 
New Hampshire 

 Dr. Sidney Darlington earned his B.S. in 
physics at Harvard, his B.S. in electrical 
communication at MIT, and his Ph.D. at 
Columbia University. In 1929 he joined 
Bell Laboratories, where he was head of 
the Circuits and Control Department. Dur-
ing that period he became good friends 
with other important contributors such as 
Edward Norton and Hendrik Bode. A 
holder of 24 U.S. patents, he was awarded 
the Presidential Medal of Freedom, the 
highest civilian honor in the United States, 
in 1945 for his contributions to network 
design during World War II. An elected 
member of the National Academy of 
Engineering, he also received the IEEE 
Edison Medal in 1975 and the IEEE 
Medal of Honor in 1981. His U.S. patent 
2 663 806 titled “Semiconductor Signal 
Translating Device” was issued on Decem-
ber 22, 1953, describing how two transis-
tors could be constructed in the Darlington 
configuration on the same substrate—
often looked upon as the beginnings of 
compound IC construction. Dr. Darlington 
was also responsible for the introduction 
and development of the Chirp technique, 
used throughout the world in waveguide 
transmission and radar systems. He is a 
primary contributor to the Bell Laborato-
ries Command Guidance System that 
guides most of the rockets used today to 
place satellites in orbit. It uses a combina-
tion of radar tracking on the ground with 
inertial control in the rocket itself. Dr. 
Darlington was an avid outdoorsman as a 
hiker and member of the Appalachian 
Mountain Club. One of his proudest 
accomplishments was being able to climb 
Mt. Washington at the age of 80.

  

  

 FIG. 5.74 

  Sidney Darlington (Courtesy of 

AT&T Archives and History Center.)   

 

V

BE

1

V

BE

2

C

1

C

2

V

i

I

B

1

I

E

2

V

o

+

+

β

β

1

 FIG. 5.75 

        Emitter-follower  configuration  with  a  Darlington  amplifier.    

305


background image

BJT AC ANALYSIS

306

 

that obtained with a single-transistor network. The current gain is also larger, but the voltage 
gain for a single-transistor or Darlington configuration remains slightly less than one. 

 

 

  DC  Bias

    The case current is determined using a modified version of Eq. 4.44. There are 

now two base-to-emitter voltage drops to include and the beta of a single transistor is 
replaced by the Darlington combination of Eq. 5.101. 

 

   I

B

1

=

V

CC

V

BE

1

V

BE

2

R

B

+ b

D

 R

E

  

(5.102)

    

 The emitter current of  Q  

1

  is equal to the base current of  Q  

2

  so that 

 

   I

E

2

= b

2

I

B

2

= b

2

I

E

1

= b

2

(

b

1

I

E

1

)

= b

1

b

2

I

B

1

   

 resulting  in 

 

   I

C

2

⬵ I

E

2

= b

D

 I

B

1

  

(5.103)    

 The collector voltage of both transistors is 

 

   V

C

1

V

C

2

V

CC

  

(5.104)    

 the emitter voltage of  Q  

2

  

 

   V

E

2

I

E

2

R

E

  

(5.105)

    

 the base voltage of  Q  

1

  

 

   V

B

1

V

CC

I

B

1

R

B

V

E

2

V

BE

1

V

BE

2

  

(5.106)    

 the collector-emitter voltage of  Q  

 

   V

CE

2

V

C

2

V

E

2

V

CC

V

E

2

  

(5.107)    

  

EXAMPLE 5.17 

  Calculate the dc bias voltages and currents for the Darlington configura-

tion  of   Fig.    5.76   . 

 

C

1

V

i

β

= 100

β

= 50

C

2

V

o

 FIG. 5.76 

         Circuit  for  Example  5.17.    


background image

307

DARLINGTON 

CONNECTION

  

Solution:  

 

 

 

   

b

D

= b

1

b

2

= (50)(100) = 5000

 

 I

B

1

=

V

CC

V

BE

1

V

BE

2

R

B

+ b

D

 R

E

=

18 V

- 0.7 V - 0.7 V

3.3 M

 + (5000)(390 )

 

 

=

18 V

- 1.4 V

3.3 M

 + 1.95 M

=

16.6 V

5.25 M

3.16 MA

 

 I

C

2

⬵ I

E

2

= b

D

 I

B

1

= (5000)(3.16 mA) = 15.80 mA

 

 V

C

1

V

C

2

18 V

 

 V

E

2

I

E

2

R

E

= (15.80 mA)(390 ) = 6.16 V

 

 V

B

1

V

E

2

V

BE

1

V

BE

2

= 6.16 V + 0.7 V + 0.7 V = 7.56 V

 

 V

CE

2

V

CC

V

E

2

= 18 V - 6.16 V = 11.84 V

      

  

AC Input Impedance 

 

 The ac input impedance can be determined using the ac equivalent 

network  of   Fig.    5.77   . 

 

Z

 

Z

i1

Z

i2

R

B

R

E

Q

2

Q

1

E

1

B

2

 FIG. 5.77 

        Finding  Z 

i

 .    

 As  defined  in   Fig.    5.77   : 

 

   Z

i

2

= b

2

(r

e

2

R

E

)

 

Z

i

1

= b

1

(r

e

1

Z

i

2

)   

 so  that  

   Z

i

1

= b

1

(r

e

1

+ b

2

(r

e

2

R

E

))   

 Assuming  

   R

E

r

e

2

   

 and  

   Z

i

1

= b

1

(r

e

1

+ b

2

R

E

)   

 Since  

   

b

2

R

E

r

e

1

 

Z

i

1

⬵ b

1

b

2

R

E

   

 and  since  

   Z

i

R

B

7

Z

i

   

 

   Z

i

R

B

7

b

1

b

2

R

E

R

B

7

b

D

 R

E

  

(5.108)    

 For  the  network  of   Fig.    5.76    

 

    Z

i

R

B

7

b

D

 R

E

 

 

= 3.3 M

7

(5000)(390 

) = 3.3 M

7

1.95 M

 

 

1.38 M

   

 Note in the preceding analysis that the values of  

e

   were not compared but dropped com-

pared to much larger quantites. In a Darlington configuration the values of  

e

   will be differ-

ent because the emitter current through each transistor will be different. Also, keep in mind 
that chances are the beta values for each transistor will be different because thay deal with 
different current levels. The fact remains, however, that the product of the two beta values 
will equal 

 D 

 , as indicated on the specification sheet.  


background image

BJT AC ANALYSIS

308

 

  AC  Current  Gain 

 

 The current gain can be determined from the equivalent network of 

 Fig.   5.78   . The output impedance of each transistor is ignored and the parameters for each 
transistor are employed. 

 

I

 

R

 

R

E

B

1

2

I

b2

B

2

E

1

C

1

C

2

E

2

I

b

I

b

I

o

1

r

e1

β

2

r

e2

β

β

1

I

b1

β

 FIG. 5.78 

        Determining  A 

i

   for  the  network  of   Fig.    5.75   .    

 Solving for the output current:       I

o

I

b

2

+ b

2

I

b

2

= (b

2

+ 1)I

b

2

   

 with  

    I

b

2

= b

1

I

b

1

I

b

1

= (b

1

+ 1)I

b

1

   

 Then  

     I

o

= (b

2

+ 1)(b

1

+ 1)I

b

1

   

 Using the current-divider rule on the input circuit: 

 

    I

b

1

=

R

B

R

B

Z

i

 I

i

=

R

B

R

B

+ b

1

b

2

R

E

 I

i

   

 and  

     I

o

= (b

2

+ 1)(b

1

+ 1)a

R

B

R

B

+ b

1

b

2

R

E

bI

i

   

 so  that  

     A

i

=

I

o

I

i

=

(

b

1

+ 1)(b

2

+ 1)R

B

R

B

+ b

1

b

2

R

E

   

 Using     

b

1

b

2

W 1    

 

   

 A

i

=

I

o

I

i

b

1

b

2

R

B

R

B

+ b

1

b

2

R

E

  

(5.109)    

 or  

   

 A

i

=

I

o

I

i

b

D

 R

B

R

B

+ b

D

 R

E

  

(5.110)

    

 For   Fig.    5.76   : 

 

    A

i

=

I

o

I

i

=

b

D

 R

B

R

B

+ b

D

 R

E

=

(5000)(3.3 M

)

3.3 M

 + 1.95 M

 

 

3.14 10

3

    

  AC  Voltage  Gain 

 

 The voltage gain can be determined using  Fig.   5.77    and the following 

derivation: 

 

   

 V

o

I

o

R

E

 

 V

i

I

i

(R

B

7

Z

i

)

 

 R

B

储 Z

i

R

B

储 b

D

 R

E

=

b

D

 R

B

 R

E

R

B

+ b

D

 R

E

   

 and  

    A

n

=

V

o

V

i

=

I

o

 R

E

I

i

(R

B

储 Z

i

)

= (A

i

)

a

R

E

R

B

Z

i

b   

 

     

=

£

b

D

 R

B

R

B

+ b

D

 R

E

§ £

R

E

b

D

 R

B

 R

E

R

B

+ b

D

 R

E

§

   

 and 

 

v

  

⬵ 1 (in reality less than one)  

(5.111) 

 

 an expected result for the emitter-follower configuration.  


background image

309

DARLINGTON 

CONNECTION

  AC  Output  Impedance 

 

 The output impedance will be determined by going back to  Fig.   5.78    

and setting  

i

   to zero volts as shown in  Fig.   5.79   . The resistor  

B

   is “shorted out,” resulting 

in  the  configuration  of   Fig.    5.80   .  Note  in   Figs.    5.82     and     5.83     that  the  output  current  has 
been redefined to match standard nomenclature and properly defined  

o

  . 

  

R

B

R

E

V

o

V

⫽ 0 V

I

o

Z

o 

I

b1

I

b2

+

1

r

e1

β

2

r

e2

β

2

I

b2

β

1

I

b1

β

 FIG. 5.79 

        Determining  Z 

o

 .    

R

E

V

o

I

b1

I

b2

I

o 

Z

o

I

e

a

+

+

+

2

r

e2

β

1

r

e1

β

1

I

b1

β

2

I

b2

β

(   

2

 

⫹ 1)I

b2

β

 FIG. 5.80 

        Redrawn  of  network  of   Fig.    5.79   .    

 At point  a  Kirchhoff’s current law will result in    I

o

+ (b

2

+ 1)I

b

2

I

e

:    

 

   I

o

I

e

- (b

2

+ 1)I

b

2

   

 Applying Kirchhoff’s voltage law around the entire outside loop will result in 

 

   

-I

b

1

b

1

r

e

1

I

b

2

b

2

r

e

2

V

o

= 0   

 and  

    V

o

I

b

1

b

1

r

e

1

I

b

2

b

2

r

e

2

   

 Substituting       I

b

2

= (b

1

+ 1)I

b

1

 

 V

o

= -I

b

1

b

1

r

e

1

- (b

1

+ 1)I

b

1

b

2

r

e

2

 

 

= -I

b

1

[

b

1

r

e

1

+ (b

1

+ 1)b

2

r

e

2

]

 

 

 

 and  

    I

b

1

= -

V

o

b

1

r

e

1

+ (b

1

+ 1)b

2

r

e

2

   

 with  

    I

b

2

= (b

1

+ 1)I

b

1

= (b

1

+ 1)c -

V

o

b

1

r

e

1

+ (b

1

+ 1)b

2

r

e

2

d

   

 so  that  

    I

b

2

= - c

b

1

+ 1

b

1

r

e

1

+ (b

1

+ 1)b

2

r

e

2

V

o

   

 Going  back  

     I

o

I

e

- (b

2

+ 1)I

b

2

I

e

- (b

2

+ 1)a-

(

b

1

+ 1)V

o

b

1

r

e

1

+ (b

1

+ 1)b

2

r

e

2

b   

 or  

     I

o

=

V

o

R

E

+

(

b

1

+ 1)(b

2

+ 1)V

o

b

1

r

e

1

+ (b

1

+ 1)b

2

r

e

2

   


background image

BJT AC ANALYSIS

310

 

 Because     

b

1

b

2

W 1    

 

    I

o

=

V

o

R

E

+

b

1

b

2

V

o

b

1

r

e

1

+ b

1

b

2

r

e

2

=

V

o

R

E

+

V

o

b

1

r

e

1

b

1

b

2

+

b

1

b

2

r

e

2

b

1

b

2

 

 I

o

=

V

o

R

E

+

V

o

r

e

1

b

2

r

e

2

   

 which defines the parallel resistance network of  Fig.   5.81   . 

 

 In  general,     R

E

W a

r

e

1

b

2

r

e

2

b    so the output impedance is defined by 

 

   Z

o

=

r

e

1

b

2

r

e

2

  

(5.112)    

 Using the dc results, the value of    r

e

2

     and     r

e

1

    can be determined as follows. 

 

    r

e

2

=

26 mV

I

E

2

=

26 mV

15.80 mA

= 1.65 ⍀

   

 and  

    I

E

1

I

B

2

=

I

E

2

b

2

=

15.80 mA

100

= 0.158 mA   

 so  that  

    r

e

1

=

26 mV

0.158 mA

= 164.5 ⍀

   

 The output impedance for the network of  Fig.   5.78    is therefore: 

 

   Z

o

r

e

1

b

2

r

e

2

=

164.5 

100

+ 1.65 ⍀ = 1.645 ⍀ + 1.65 ⍀ = 3.30 ⍀   

 In general, the output impedance for the configuration of  Fig.   5.78    is very low—in the 
order of a few ohms at most.  

  Voltage-Divider  Amplifier 

  

DC Bias    

  Let us now investigate the effect of the Darlington configuration in a basic 

amplifier configuration as shown in  Fig.   5.82   . Note that now there is a collector resistor 
 

C

  , and the emitter terminal of the Darlington circuit is connected to ground for ac condi-

tions. As noted on  Fig.   5.82   , the beta of each transistor is provided along with the resulting 
voltage from base to emitter.  

R

E

V

o

I

o

Z

o 

+

r

e1

2

r

e2

 FIG. 5.81 

        Resulting  network  defined  by  Z 

o

 .    

V

i

V

o

I

o

C

1

R

2 

 

R

E

R

C

Darlington

Pair

C

E

C

2

V

BE

 = 1.5 V

R

1 

 

Z

i

I

i

220 k

Ω

  

680 

Ω

  

470 k

Ω

  

1.2 k

Ω

  

V

CC

 = 27 V

 1

 =    

2

 = 110.

Z

i

'

β

β

 FIG. 5.82 

        Amplifier  configuration  using  a  Darlington  pair.    


background image

311

DARLINGTON 

CONNECTION

 The dc analysis can proceed as follows: 

 

   

b

D

= b

1

b

2

= (110 * 110) = 12,100

 

V

B

=

R

2

R

2

R

1

V

CC

=

220 k

(27 V)

220 k

 + 470 k

8.61 V

 

 V

E

V

B

V

BE

= 8.61 V - 1.5 V = 7.11 V

 

 I

E

=

V

E

R

E

=

7.11 V

680 

10.46 mA

 

 I

B

=

I

E

b

D

=

10.46 mA

12,100

0.864 MA

   

 Using the preceding results the values of    r

e

2

     and     r

e

1

     can  be  determined: 

 

   r

e

2

=

26 mV

I

E

2

=

26 mV

10.46 mA

2.49 ⍀

 

 I

E

1

I

B

2

=

I

E

2

b

2

=

10.46 mA

110

= 0.095 mA   

 and  

    r

e

1

=

26 mV

I

E

1

=

26 mV

0.095 mA

273.7 ⍀

     

  AC  Input  Impedance 

 

 

The  ac  equivalent  of   Fig.    5.82     appears  as   Fig.    5.83   .  The  resistors   R  

1

  

and  R  

2

  are in parallel with the input impedance to the Darlington pair, assuming the second 

transistor found by assuming the second transistor acts like an  

E

   load on the first as 

shown  in   Fig.    5.83   . 

 That  is,     Z

i

= b

1

r

e

1

+ b

1

(

b

2

r

e

2

)    

 

V

i

V

o

I

o

R

1

R

2

   2

r

e

2

R

C

I

i

Q

1 

 

Q

2

 

Z

i

Z

i

'

β

   2

r

e

2

β

Q

1 

 

 FIG. 5.83 

        Defining      Z

i

     and   Z 

i

 .    

 and  

    Z

i

 = b

1

[r

e

1

+ b

2

r

e

2

]  

(5.113)    

 For  the  network  of   Fig.    5.82   : 

 

    Z

i

 = 110[273.7  + (110)(2.49 )]

 

 

= 110[273.7  + 273.9 ]

 

 

= 110[547.6 ]

 

 

60.24 k

   

 and  

     Z

i

R

1

7

R

2

7

Z

i

 

 

= 470 k

7

220 k

7

60.24 k

 

 

= 149.86 k

7

60.24 k

 

 

42.97 k

    


background image

BJT AC ANALYSIS

312

 

  AC  Current  Gain 

 

 The  complete  ac  equivalent  of   Fig.    5.82     appears  as   Fig.    5.84.    

 

Z

i

I

b1

I

b2

R

1 

 

R

2 

 

B

1 

 

E

2 

 

E

2 

 

1

r

e1

E

1

B

2 

 

C

1

C

2 

 

R

C

I

 

I

o

V

o

Z

i

'

I

i

'

β

1

I

b1

β

2

I

b2

β

2

r

e2

β

 FIG. 5.84 

        ac  equivalent  network  for   Fig.    5.82   .    

 The  output  current  

     I

o

= b

1

I

b

1

+ b

2

I

b

2

   

 with  

    I

b

2

= (b

1

+ 1)I

b

1

   

 so  that  

     I

o

= b

1

I

b

1

+ b

2

(

b

1

+ 1)I

b

1

   

 and  with  

    I

b

1

I

i

   

 we  find  

     I

o

= b

1

I

i

 + b

2

(

b

1

+ 1)I

i

    

 and  

    A

i

 =

I

o

I

i

= b

1

+ b

2

(

b + 1)

 

 

⬵ b

1

+ b

2

b

1

= b

1

(1

+ b

2

)

 

 

⬵ b

1

b

2

   

 and  finally  

   A

i

 =

I

o

I

i

= b

1

b

2

= b

D

  

(5.114)    

 For  the  original  structure:  

    I

i

 =

R

1

R

2

I

i

R

1

R

2

Z

i

     or     

I

i

I

i

=

R

1

R

2

R

1

R

2

Z

i

    

 but  

   A

i

=

I

o

I

i

= a

I

o

I

i

b a

I

i

I

i

b   

 so  that  

    A

i

=

b

D

(R

1

R

2

)

R

1

R

2

Z

i

  

(5.115)    

 For   Fig.    5.82     

    A

i

=

(12,100)(149.86 k

)

149.86 k

 + 60.24 k

 

 

8630.7

   

 Note the significant drop in current gain due to  R  

1

  and  R  

2

 .  

  AC  Voltage  Gain 

 

 The input voltage is the same across  R  

1

  and  R  

2

  and at the base of the 

first transistor as shown in  Fig.    5.84   . 

 The result is 

 

   A

v

=

V

o

V

i

= -

I

o

 R

C

I

i

Z

i

= -A

i

a

R

C

Z

i

b   

 and  

   A

v

= -

b

D

R

C

Z

i

  

(5.116)    

 For  the  network  of   Fig.    5.82   , 

 

   A

v

= -

b

D

R

C

Z

i

= -

(12,000)(1.2 k

)

60.24 k

= ⴚ241.04    


background image

313

DARLINGTON 

CONNECTION

  AC  Output  Impedence 

 

 Because the output impedance in  

C

   is parallel with the collector 

to emitter terminals of the transistor, we can look back on similar situations and find that 
the output impedance is defined by 

 

   Z

o

⬵ R

C

7

r

o

2

  

(5.117)    

 where     r

o

2

    is the output resistance of the transistor Q 

2

 .  

  Packaged  Darlington  Amplifier 

 Because the Darlington connection is so popular, a number of manufacturers provide 
packaged units such as shown in  Fig.   5.85   . Typically, the two BJTs are constructed on a 
single chip rather than separate BJT units. Note that only one set of collector, base, and 
emitter terminals is provided for each configuration. These, of course, are the base of the 
transistor  Q  

1

 , the collector of  Q  

1

  and  Q  

2

 , and the emitter of  Q  

2

 .       

  

C

B E

(a)

  

      

C

B

E

(b)

 FIG. 5.85 

     

  Packaged Darlington amplifiers: (a) TO-92 package; 

(b) Super SOT™-3 package.  

 

 In  Fig.   5.86    some of the ratings for an MPSA28 Fairchild Semiconductor Darlington 

amplifier are provided. In particular, note that the maximum collector-to-emitter voltage of 
80 V is also the breakdown voltage. The same is true for the collector-to-base and emitter-
to-base voltages, although notice how much lower the maximum ratings are for the base-
to-emitter junction. Because of the Darlington configuration, the maximum current rating 
for the collector current has jumped to 800 mA—far exceeding levels we have encountered 

 FIG. 5.86 

  MPSA  28  Fairchild  Semiconductor  Darlington  amplifier  ratings.        

 Absolute Maximum Ratings 

  

CES

   

 Collector-Emitter  Voltage 

 80  V 

  

CBO

   

 Collector-Base  Voltage 

 80  V 

  

EBO

   

 Emitter-Base  Voltage 

 12  V 

  

C

   

 Collector  Current–Continuous 

 800  mA 

 Electrical  Characteristics 

  

(BR)CES

   

 Collector-Emitter Breakdown Voltage 

 80 V 

  

(BR)CBO

   

 Collector-Base Breakdown Voltage 

 80 V 

  

(BR)EBO

   

 Emitter-Base Breakdown Voltage 

 12 V 

  

CBO

   

 Collector Cutoff Current 

 100 mA 

  

EBO

   

 Emitter Cutoff Current 

 100 mA 

 On  Characteristics 

  

FE

   

 DC Current Gain 

 10,000 

  

CE(sat)

   

 Collector-Emitter Saturation Voltage 

 1.2 V 

  

BE(on)

   

 Base-Emitter on Voltage 

 2.0  V 


background image

BJT AC ANALYSIS

314

 

for single-transistor networks. The dc current gain is rated at the high level of 10,000 and 
the base-to-emitter potential in the “on” state is 2 V, which certainly exceeds the 1.4 V we 
have used for individual transistors. Finally, it is interesting to note that the level of  

CEO

   is 

much higher at 500 nA than for a typical single-transistor unit.     

 In the packaged format the network of  Fig.   5.75    would appear as shown in  Fig.   5.87   . 

Using 

 

D

   and the provided value of    V

BE

 (

=V

BE

1

V

BE

2

),    all the equations appearing in 

this section can be applied. 

 

  

β

D  

= 10,000 

V

BE  

= 2.0 V

V

o

V

i

C

2

R

E

390 

Ω

MPSA 28 Darlington Amplifier

C

1

B

C

E

R

B

3.3 M

Ω

+V

CC

(

+18 V)

I

i

I

o

Z

i

 FIG. 5.87 

        Darlington  emitter-follower  circuit.   

   5.18 

 FEEDBACK  PAIR 

 ● 

 The feedback pair connection (see  Fig.   5.88   ) is a two-transistor circuit that operates like 
the Darlington circuit. Notice that the feedback pair uses a  pnp  transistor driving an  npn  
transistor, the two devices acting effectively much like one  pnp  transistor. As with a Dar-
lington connection, the feedback pair provides very high current gain (the product of the 
transistor current gains), high input impedance, low output impedance, and a voltage gain 
slightly less than one. Initially, it may appear that it would have a high voltage gain because 
the output is taken off the collector with a resistor  

C

   in place. However, the pnp–npn 

combination results in terminal characteristics very similar to that of the emitter–follower 
configuration. A typical application (see  Chapter   12   ) uses a Darlington and a feedback-pair 
connection to provide complementary transistor operation. A practical network employing a 
feedback pair is provided in  Fig.   5.89    for investigation.   

  DC  Bias 

 The dc bias calculations that follow use practical simplifications wherever possible to pro-
vide simpler results. From the  Q  

1

  base–emitter loop, one obtains 

 

   V

CC

I

C

R

C

V

EB

1

I

B

1

R

B

= 0

 

V

CC

- (b

1

b

2

I

B

1

)R

C

V

EB

1

I

B

1

R

B

= 0   

 The base current is then 

 

   I

B

1

=

V

CC

V

BE

1

R

B

+ b

1

b

2

R

C

  

(5.118)    

 The collector current of  Q  

1

   is 

 

   I

C

1

= b

1

I

B

1

I

B

2

   

 which is also the base  Q  

2

  current. The transistor  Q  

2

  collector current is 

 

   I

C

2

= b

2

I

B

2

⬇ I

E

2

   

 FIG. 5.88 

        Feedback  pair  connection.    


background image

315

FEEDBACK PAIR

V

i

V

o

 FIG. 5.89 

        Operation  of  a  feedback  pair.    

 so that the current through  

C

   is 

 

   I

C

I

E

1

I

C

2

⬇ I

B

2

I

C

2

    

(5.119)

 The  voltages 

V

C

2

V

E

1

V

CC

I

C

R

C

 

(5.120)

and  

   V

B

1

I

B

1

R

B

  

(5.121)

    

 with  

   V

BC

1

V

B

1

V

BE

2

V

B

1

- 0.7 V  

(5.122)

    

  

EXAMPLE 5.18 

  Calculate the dc bias currents and voltages for the circuit of  Fig.   5.89    to 

provide  

o

   at one-half the supply voltage (9 V). 

  

Solution:  

 

 

 

   I

B

1

=

18 V

- 0.7 V

2 M

 + (140)(180)(75 )

=

17.3 V

3.89

* 10

6

4.45 MA   

 The base  Q  

2

  current is then 

 

   I

B

2

I

C

1

= b

1

I

B

1

= 140(4.45 mA) = 0.623 mA   

 resulting in a  Q  

2

  collector current of 

 

   I

C

2

= b

2

I

B

2

= 180(0.623 mA) = 112.1 mA   

 and the current through  

C

   is then 

 Eq.  (5.119):     I

C

I

E

1

I

C

2

= 0.623 mA + 112.1 mA ⬇ I

C

2

112.1 mA    

    

 V

C

2

V

E

1

= 18 V - (112.1 mA)(75 )

 

 

= 18 V - 8.41 V

 

 

9.59 V

 

 V

B

1

I

B

1

R

B

= (4.45 mA)(2 M)

 

 

8.9 V

 

 V

BC

1

V

B

1

- 0.7 V = 8.9 V - 0.7 V

 

 

8.2 V

      


background image

BJT AC ANALYSIS

316

 

  AC  Operation 

 The ac equivalent circuit for that of  Fig.   5.89    is drawn in  Fig.   5.90   . 

 

I

b1

a

I

b2

R

B

R

C

1

r

e1

I

 

I

i

Z

i

V

i

+

V

o

+

I

i

'

β

2

r

e2

β

1

I

b1

β

2

I

b2

β

Z

i

'

 FIG. 5.90 

          ac  equivalent  for  the  network  of   Fig.    5.89   .    

  

Input Impedance,  Z 

i

 

     

  The ac input impedance seen looking into the base of transistor  Q  

1

  

is determined as follows: 

   Z

i

 =

V

i

I

i

   

 Applying Kirchhoff’s current law at node  a  and defining    I

c

I

o

   : 

   I

b

1

+ b

1

I

b

1

- b

2

I

b

2

I

o

= 0   

 with     I

b

2

= -b

1

I

b

1

     as  noted  in   Fig.    5.90   . 

 The  result  is 

    I

b

1

+ b

1

I

b

1

- b

2

(

-b

1

I

b

1

)

I

o

= 0   

 and 

    I

o

= -I

b

1

- b

1

I

b

1

- b

1

b

2

I

b

1

   

 or 

    I

o

= -I

b

1

(1

+ b

1

)

- b

1

b

2

I

b

1

   

 but 

    

b

1

W 1   

 and 

     I

o

= -b

1

I

b

1

- b

1

b

2

I

b

1

= -I

b

1

(

b

1

+ b

1

b

2

)

 

 

= -I

b

1

b

1

(1

+ b

2

)

 

 

 

 resulting  in: 

    I

o

⬵ -b

1

b

2

I

b

1

  

(5.123)    

 Now,     I

b

1

=

V

i

V

o

b

1

r

e

1

     from   Fig.    5.90    

 and 

    V

o

= -I

o

 R

C

= -(-b

1

b

2

I

b

1

)R

C

= b

1

b

2

I

b

1

R

C

   

 so 

    I

b

1

=

V

i

- b

1

b

2

I

b

1

R

C

b

1

r

e

1

   

 Rearranging: 

    I

b

1

b

1

r

e

1

V

i

- b

1

b

2

I

b

1

R

C

   

 and 

    I

b

1

(

b

1

r

e

1

+ b

1

b

2

R

C

)

V

i

   

 so 

    I

b

1

I

i

 =

V

i

b

1

r

e

1

+ b

1

b

2

R

C

   

 and 

    V

i

=

V

i

I

i

=

V

i

V

i

b

1

r

e

+ b

1

b

2

R

C

   

 so  that 

    Z

i

 = b

1

r

e

1

+ b

1

b

2

R

C

  

(5.124)

    

 In  general, 

    

b

1

b

2

R

C

W b

1

r

e

1

   

 and 

    Z

i

 ⬵ b

1

b

2

R

C

  

(5.125)    


background image

317

FEEDBACK PAIR

 with 

    Z

i

R

B

7

Z

i

  

(5.126)

    

 For  the  network  of   Fig.    5.89   : 

    r

e

1

=

26 mV

I

E

1

=

26 mV

0.623 mA

= 41.73    

 and 

     Z

i

 = b

1

r

e

1

+ b

1

b

2

R

C

 

 

= (140)(41.73 ) + (140)(180)(75 )

 

 

= 5842.2  + 1.89 M

 

 

1.895 M

   

 where Eq. (5.125) results in    Z

i

 ⬵ b

1

b

2

R

C

= (140)(180)(75 ) = 1.89 M⍀,     validating 

the above approximations.   

  Current  Gain 

 Defining     I

b

1

I

i

    as shown in  Fig.   5.90    will permit finding the current gain    A

i

 = I

o

>I

i

.    

 Looking back on the derivation of  

i

   we found    I

o

= -b

1

b

2

I

b

1

= -b

1

b

2

I

i

    

 resulting  in 

    A

i

 =

I

o

I

i

= -b

1

b

2

  

(5.127)    

 The current gain    A

i

I

o

>I

i

    can be determined using the fact that 

   A

i

=

I

o

I

i

=

I

o

I

i

#

I

i

I

i

   

 For  the  input  side: 

    I

i

 =

R

B

I

i

R

B

Z

i

=

R

B

I

i

R

B

+ b

1

b

2

R

C

   

 Substituting: 

    A

i

=

I

o

I

i

#

I

i

I

i

= (-b

1

b

2

)

a

R

B

R

B

+ b

1

b

2

R

C

b   

 So  that 

    A

i

=

I

o

I

i

=

-b

1

b

2

R

B

R

B

+ b

1

b

2

R

C

  

(5.128)    

 The negative sign appears because both  

i

   and  

o

   are defined as entering the network. 

 For  the  network  of   Fig.    5.89   : 

     A

i

 =

I

o

I

i

= -b

1

b

2

 

 

= -(140)(180)

 

 

= ⴚ25.2 10

3

 

 A

i

=

-b

1

b

2

R

B

R

B

+ b

1

b

2

R

c

= -

(140)(180)(2 M

)

2 M

 + 1.89 M

 

 

= -

50,400 M

3.89 M

 

 

= ⴚ12.96 10

3

 (

⬵ half of A

i

)

 

 

 

 

  Voltage  Gain 

 The voltage gain can quickly be determined using the results obtained above. 

 That  is, 

     A

v

=

V

o

V

i

=

-I

o

R

C

I

i

Z

i

 

 

= -

(

-b

1

b

2

I

i

)R

C

I

i

(b

1

r

e

1

+ b

1

b

2

R

C

)

   

    

A

v

=

b

2

R

C

r

e

1

+ b

2

R

C

  

(5.129)

    


background image

BJT AC ANALYSIS

318

 

 which is simply the following if we apply the approximation:    

b

2

R

C

r

e

1

    

    

A

v

b

2

R

C

b

2

R

C

= 1   

 For  the  network  of   Fig.    5.89   : 

    A

v

=

b

2

R

C

r

e

1

+ b

2

R

C

=

(180)(75 

)

41.73 

 + (180)(75 )

 

 

=

13.5

* 10

3

 

41.73 

 + 13.5 * 10

3

 

 

 

0.997 ⬵ 1 (as indicated above)    

  Output  Impedance 

 The output impedance    Z

o

     is  defined  in   Fig.    5.91     when   

i

   is set to zero volts. 

 

I

b2

R

C

I

o

Z

o

I

b1

+

V

o

V

o

+

1

r

e1

β

2

r

e2

β

1

I

b1

β

2

I

b2

β

Z

o

'

 FIG. 5.91 

          Determining  Z 

o

    and Z 

o

 .    

 Using the fact that    I

o

= -b

1

b

2

I

b

1

    from calculations above, we find that 

    

 Z

o

 =

V

o

I

o

=

V

o

-b

1

b

2

I

b

1

   

 but 

     I

b

1

= -

V

o

b

1

r

e

1

   

 and 

     Z

o

 =

V

o

-b

1

b

2

a-

V

o

b

1

r

e

1

b

=

b

1

r

e

1

b

1

b

2

   

 so  that 

    Z

o

 =

r

e

1

b

2

  

(5.130

)    

 with 

    Z

o

R

C

g

r

e

1

b

2

  

(5.131)    

 However, 

    R

C

W

r

e

1

b

2

   

 leaving 

    Z

o

r

e

1

b

2

  

(5.132)    

 which will be a very low value. 
 For  the  network  of   Fig.    5.89   : 

    

Z

o

41.73 

180

0.23 ⍀   

 The preceding analysis shows that the feedback pair connection of  Fig.   5.89    provides 

operation with voltage gain very near 1 ( just as with a Darlington emitter-follower), a very 
high current gain, a very low output impedance, and a high input impedance.   


background image

319

THE HYBRID 

EQUIVALENT MODEL

   5.19 

 THE  HYBRID  EQUIVALENT  MODEL 

 ● 

 The hybrid equivalent model was mentioned in the earlier sections of this chapter as one 
that was used in the early years before the popularity of the  

e

   model developed. Today 

there is a mix of usage depending on the level and direction of the investigation. 

  The  r 

e

  model has the advantage that the parameters are defined by the actual operating 

conditions,  

 whereas 

  the parameters of the hybrid equivalent circuit are defined in general terms for any 
operating conditions.  

 In other words, the hybrid parameters may not reflect the actual operating conditions 

but simply provide an indication of the level of each parameter to expect for general use. 
The  

e

   model suffers from the fact that parameters such as the output impedance and the 

feedback elements are not available, whereas the hybrid parameters provide the entire set 
on the specification sheet. In most cases, if the  

e

   model is employed, the investigator will 

simply examine the specification sheet to have some idea of what the additional elements 
might be. This section will show how one can go from one model to the other and how the 
parameters are related. Because all specification sheets provide the hybrid parameters and 
the model is still extensively used, it is important to be aware of both models. The hybrid 
parameters as shown in  Fig.   5.92    are derived from the specification sheet for the 2N4400 
transistor described in  Chapter   3   . The values are provided at a dc collector current of 1 mA 
and a collector-to-emitter voltage of 10 V. In addition, a range of values is provided for 
each parameter for guidance in the initial design or analysis of a system. One obvious ad-
vantage of the specification sheet listing is the immediate knowledge of typical levels for 
the parameters of the device as compared to other transistors. 

 

 FIG. 5.92 

          Hybrid  parameters  for  the  2N4400  transistor.    

 FIG. 5.93 

          Two-port  system.    

 The description of the hybrid equivalent model will begin with the general two-port 

system of  Fig.   5.93   . The following set of equations (5.131) and (5.132) is only one of 
a number of ways in which the four variables of  Fig.   5.93    can be related. It is the most 
frequently employed in transistor circuit analysis, however, and therefore is discussed in 
detail in this chapter. 

 


background image

BJT AC ANALYSIS

320

 

    

V

i

h

11

I

i

h

12

V

o

  

(5.133)

    

    

I

o

h

21

I

i

h

22

V

o

  

(5.134)    

 The parameters relating the four variables are called  h-parameters , from the word 

“hybrid.” The term  hybrid  was chosen because the mixture of variables ( V  and   ) in each 
equation results in a “hybrid” set of units of measurement for the  h -parameters. A clearer 
understanding of what the various  h -parameters represent and how we can determine their 
magnitude can be developed by isolating each and examining the resulting relationship. 

      h  

11  

   

  If we arbitrarily set    V

o

= 0    (short circuit the output terminals) and solve for  h  

11

   in 

Eq. (5.133), we find 

    

h

11

=

V

i

I

i

`

V

o

=0

    ohms  

(5.135)    

 The ratio indicates that the parameter  h  

11

  is an impedance parameter with the units of ohms. 

Because it is the ratio of the  input  voltage to the  input  current with the output terminals 
 shorted , it is called the  short-circuit input-impedance parameter.  The subscript 11 of  h  

11

  

refers to the fact that the parameter is determined by a ratio of quantities measured at the 
input terminals.  

   h  

12

     

  If   

i

   is set equal to zero by opening the input leads, the following results for  h  

12

 : 

    

h

12

=

V

i

V

o

`

I

l

=0

    unitless  

(5.136)

    

 The parameter  h  

12

 , therefore, is the ratio of the input voltage to the output voltage with 

the input current equal to zero. It has no units because it is a ratio of voltage levels and is 
called the  open-circuit reverse transfer voltage ratio parameter.  The subscript 12 of  h  

12

  

indicates that the parameter is a transfer quantity determined by a ratio of input (1) to out-
put (2) measurements. The first integer of the subscript defines the measured quantity to 
appear in the numerator; the second integer defines the source of the quantity to appear in 
the denominator. The term  reverse  is included because the ratio is an input voltage over an 
output voltage rather than the reverse ratio typically of interest.  

   h  

21  

   

  If in Eq. (5.134)  

o

   is set equal to zero by again shorting the output terminals, the 

following results for  h  

21

 : 

    

h

21

=

I

o

I

i

`

V

o

=0

    unitless  

(5.137)

    

 Note that we now have the ratio of an output quantity to an input quantity. The term  forward  
will now be used rather than  reverse  as indicated for  h  

12

 . The parameter    h

21

    is the ratio of 

the output current to the input current with the output terminals shorted. This parameter, 
like  h  

12

 , has no units because it is the ratio of current levels. It is formally called the  short-

circuit forward transfer current ratio parameter.  The subscript 21 again indicates that it 
is a transfer parameter with the output quantity (2) in the numerator and the input quantity 
(1) in the denominator.  

   h  

22

     

  The last parameter,  h  

22

 , can be found by again opening the input leads to set    I

1

= 0    

and solving for  h  

22

  in Eq. (5.134): 

    

h

22

=

I

o

V

o

`

I

i

=0

    siemens  

(5.138)    

 Because it is the ratio of the output current to the output voltage, it is the output conductance 
parameter, and it is measured in siemens (S). It is called the  open-circuit output admittance 
parameter.
  The subscript 22 indicates that it is determined by a ratio of output quantities. 


background image

321

THE HYBRID 

EQUIVALENT MODEL

 Because each term of Eq. (5.133) has the unit volt, let us apply Kirchhoff’s voltage law 

“in reverse” to find a circuit that “fits” the equation. Performing this operation results in 
the  circuit  of   Fig.    5.94   .  Because  the  parameter   h  

11

  has the unit ohm, it is represented by a 

resistor  in   Fig.    5.94   .  The  quantity   h  

12

  is dimensionless and therefore simply appears as a 

multiplying factor of the “feedback” term in the input circuit. 

 Because each term of Eq. (5.134) has the units of current, let us now apply Kirchhoff’s 

current law “in reverse” to obtain the circuit of  Fig.   5.95   . Because  h  

22

  has the units of 

admittance, which for the transistor model is conductance, it is represented by the resistor 
symbol. Keep in mind, however, that the resistance in ohms of this resistor is equal to the 
reciprocal of conductance (1

兾 h  

22

 ). 

 The complete “ac” equivalent circuit for the basic three-terminal linear device is indi-

cated in  Fig.   5.96    with a new set of subscripts for the  h -parameters. The notation of  Fig. 
  5.96    is of a more practical nature because it relates the  h -parameters to the resulting ratio ob-
tained in the last few paragraphs. The choice of letters is obvious from the following listing: 

    

h

11

input resistance S h

i

 

h

12

reverse transfer voltage ratio S h

r

 

h

21

forward transfer current ratio S h

f

 

h

22

output conductance S h

o

   

   

 FIG. 5.94 

          Hybrid  input  equivalent  circuit.    

 FIG. 5.95 

          Hybrid  output  equivalent  circuit.    

+

 FIG. 5.96 

          Complete  hybrid  equivalent  circuit.    

 The circuit of  Fig.   5.96    is applicable to any linear three-terminal electronic device or system 
with no internal independent sources. For the transistor, therefore, even though it has 
three basic configurations,  they are all three-terminal configurations , so that the resulting 
equivalent circuit will have the same format as shown in  Fig.   5.96   . In each case, the bottom 
of the input and output sections of the network of  Fig.   5.96    can be connected as shown in 
 Fig.   5.97    because the potential level is the same. Essentially, therefore, the transistor model 
is a three-terminal two-port system. The  h -parameters, however, will change with each 
configuration. To distinguish which parameter has been used or which is available, a second 

 FIG. 5.97 

        Common-emitter  configuration:  (a)  graphical  symbol;  (b)  hybrid  equivalent  circuit.    


background image

BJT AC ANALYSIS

322

 

subscript has been added to the  h -parameter notation. For the common-base configuration, 
the lowercase letter  b  was added, whereas for the common-emitter and common-collector 
configurations, the letters  e  and  c  were added, respectively. The hybrid equivalent network 
for  the  common-emitter  configuration  appears  with  the  standard  notation  in   Fig.    5.97   .  Note 
that     I

i

I

b

I

o

I

c

,    and, through an application of Kirchhoff’s current law,    I

e

I

b

I

c

.    

The input voltage is now  

be

  , with the output voltage    V

ce

.    For the common-base configura-

tion  of   Fig.    5.98   ,     I

i

I

e

,       I

o

I

c

     with     V

eb

V

i

     and     V

cb

V

o

.     The  networks  of   Figs.    5.97    

and     5.98     are  applicable  for   pnp  or  npn   transistors. 

  

 FIG. 5.98 

        Common-base  configuration:  (a)  graphical  symbol;  (b)  hybrid  equivalent  circuit.    

h

i

I

i

+

I

o

+

V

o

V

V

i

V

I

i

h

f

 FIG. 5.99 

        Effect  of  removing  h  

re

    and  h  

oe

   from the hybird 

equivalent  circuit.    

 FIG. 5.100 

        Approximate  hybrid  equivalent  model.    

 The fact that both a Thévenin and a Norton circuit appear in the circuit of  Fig.   5.96    was 

further impetus for calling the resultant circuit a  hybrid  equivalent circuit. Two additional 
transistor equivalent circuits, not to be discussed in this text, called the  z -parameter  and 
 y -parameter equivalent circuits, use either the voltage source or the current source, but not 
both, in the same equivalent circuit. In  Appendix   A    the magnitudes of the various param-
eters will be found from the transistor characteristics in the region of operation resulting in 
the desired  small-signal equivalent network  for the transistor. 

 For the common-emitter and common-base configurations, the magnitude of  

r

   and  

o

   

is often such that the results obtained for the important parameters such as  

i

  ,   

o

  ,   

v

  ,  and 

 

i

   are only slightly affected if  

r

   and  

o

   are not included in the model. 

 Because   

r

   is normally a relatively small quantity, its removal is approximated by 

   h

r

⬵ 0     and     h

r

V

o

= 0,    resulting in a short-circuit equivalent for the feedback element as 

shown  in   Fig.    5.99   .  The  resistance  determined  by     1

>h

o

    is often large enough to be ignored 

in comparison to a parallel load, permitting its replacement by an open-circuit equivalent 
for the CE and CB models, as shown in  Fig.    5.99   . 

 The resulting equivalent of  Fig.   5.100    is quite similar to the general structure of the 

common-base and common-emitter equivalent circuits obtained with the  

e

   model. In fact, 


background image

323

THE HYBRID 

EQUIVALENT MODEL

the hybrid equivalent and the  

e

   models for each configuration are repeated in  Fig.   5.101    

for comparison. It should be reasonably clear from  Fig.    5.101a     that 

    

h

ie

= br

e

  

(5.139)    

 and 

    h

fe

= b

ac

  

(5.140)

    

  

 From   Fig.    5.101b   , 

    

h

ib

r

e

  

(5.141)    

 and 

    h

fb

= -a  ⬵ -1  

(5.142)    

 In particular, note that the minus sign in Eq. (5.142) accounts for the fact that the current 
source of the standard hybrid equivalent circuit is pointing down rather than in the actual 
direction as shown in the  

e

   model  of   Fig.    5.101b   . 

 

I

b

h

fe

h

ie

b

e

I

b

I

c

c

e

r

e

b

e

I

b

I

c

c

e

b

I

e

b

I

e

c

e

I

c

r

e

e

I

α

I

b

h

f b

h

ib

e

b

I

e

I

c

c

b

(a)

(b)

β

β

 FIG. 5.101 

        Hybrid  versus   r 

e

   model: (a) common-emitter configuration; (b) common-base configuration.    

  

EXAMPLE 5.19 

  Given     I

E

= 2.5 mA, h

fe

= 140, h

oe

= 20 mS (mmho),     and     h

ob

= 0.5 mS,    

determine: 

     a.    The  common-emitter  hybrid  equivalent  circuit.  
    b.    The  common-base   

e

   model.   

  

Solution:  

 

 

    a.        r

e

=

26 mV

I

E

=

26 mV

2.5 mA

10.4 ⍀    

        h

ie

= br

e

= (140)(10.4 ) = 1.456 k

     r

o

=

1

h

oe

=

1

20 

mS

50 k⍀   


background image

BJT AC ANALYSIS

324

 

 Note   Fig.    5.102   .  

 FIG. 5.103 

          Common-base   r 

e

    model  for  the  parameters  of   Example    5.19   .    

I

b

h

ie

b

e

I

b

k

Ω

1.456

h

oe

1

140

c

e

= 50 k

Ω

 FIG. 5.102 

          Common-emitter  hybrid  equivalent  circuit  for  the  parameters  of   Example    5.19   .    

   b.        r

e

10.4 ⍀

    

    

 

a ⬵ 1,  r

o

=

1

h

ob

=

1

0.5 

mS

2 M⍀   

 Note   Fig.    5.103   .   

  

  

 A series of equations relating the parameters of each configuration for the hybrid 

equivalent  circuit  is  provided  in   Appendix    B   .  In   Section    5.23     it  is  demonstrated  that  the 
hybrid  parameter     h

fe

 (

b

ac

)    is the least sensitive of the hybrid parameters to a change in col-

lector current. Assuming, therefore, that    h

fe

= b    is a constant for the range of interest, is 

a fairly good approximation. It is    h

ie

= br

e

    that will vary significantly with  

C

   and should 

be determined at operating levels because it can have a real effect on the gain levels of a 
transistor amplifier.    

   5.20 

 APPROXIMATE  HYBRID  EQUIVALENT  CIRCUIT 

 ● 

 The analysis using the approximate hybrid equivalent circuit of  Fig.   5.104    for the common-
emitter configuration and of  Fig.   5.105    for the common-base configuration is very similar 
to that just performed using the  

e

   model. A brief overview of some of the most important 

configurations will be included in this section to demonstrate the similarities in approach 
and the resulting equations. 

  

h

ie

I

b

B

C

E

E

h

fe

I

b

1/h

oe

 FIG. 5.104 

          Approximate  common-emitter  hybrid  equivalent  circuit.    

E             

C

B

B

h

f b

I

e

h

ib

e

I

1/h

ob

 FIG. 5.105 

          Approximate  common-base  hybrid  equivalent  circuit.    


background image

325

APPROXIMATE HYBRID 

EQUIVALENT CIRCUIT

 Because the various parameters of the hybrid model are specified by a data sheet or 

experimental analysis, the dc analysis associated with use of the  

e

   model is not an integral 

part of the use of the hybrid parameters. In other words, when the problem is presented, the 
parameters such as  

ie

  ,   

fe

  ,   

ib

  , and so on, are specified. Keep in mind, however, that the 

hybrid parameters and components of the  

e

   model are related by the following equations, as 

discussed earlier in this chapter:    h

ie

= br

e

,       h

fe

= b,       h

oe

= 1>r

o

,       h

f b

= -a,     and     h

ib

r

e

.    

  Fixed-Bias  Configuration 

 For the fixed-bias configuration of  Fig.   5.106   , the small-signal ac equivalent network will 
appear as shown in  Fig.   5.107    using the approximate common-emitter hybrid equivalent 
model.  Compare  the  similarities  in  appearance  with   Fig.    5.22     and  the   

e

   model analysis. 

The similarities suggest that the analyses will be quite similar, and the results of one can be 
directly related to the other. 

  

R

B

V

CC

C

2  

h

fe

C

1  

R

C

h

ie

V

i

Z

o 

+

Z

 

V

 

+

I

o

I

i

 FIG. 5.106 

          Fixed-bias  configuration.    

V

i

Z

o 

+

V

 

+

R

C

h

ie

R

B

I

b 

I

o

h

fe

I

b

I

c

Z

 

I

i

1/h

oe

 FIG. 5.107 

         Substituting the approximate hybrid equivalent circuit into the ac 

equivalent  network  of   Fig.    5.106   .    

  

i

     

  From   Fig.    5.107   , 

    

Z

i

R

B

7

h

ie

  

(5.143)     

  

o

     

  From   Fig.    5.107   , 

    

Z

o

R

C

7

1

>h

oe

  

(5.144)

     

  

v

     

  Using     R

 = 1>h

oe

7

R

C

,     we  obtain 

    

 V

o

= -I

o

 R

 = -I

C

R

 

 

= -h

fe

 I

b

 R

   

 and 

    I

b

=

V

i

h

ie

   

 with 

    V

o

= -h

fe

 

V

i

h

ie

R

   

 so  that 

    A

v

=

V

o

V

i

= -

h

ie

(R

C

7 1>h

oe

)

h

ie

  

(5.145)

     

  

i

     

  Assuming  that     R

B

h

ie

     and     1

>h

oe

Ú 10R

C

,     we  find     I

b

⬵ I

i

     and     I

o

I

c

=

      h

fe

I

b

h

fe

I

i

,     and  so 

    

A

i

=

I

o

I

i

⬵ h

fe

  

(5.146)    


background image

BJT AC ANALYSIS

326

 

  

EXAMPLE 5.20 

  For  the  network  of   Fig.    5.108   ,  determine: 

    a.      

i

  .  

    b.       

o

  .  

    c.       

v

  .  

    d.       

i

  .   

 

V

i

V

 

h

fe

= 120

1.175 k

Ω

h

ie

=

20    A/V

μ

h

oe

=

2.7 k

Ω

8 V

330 k

Ω

I

o

Z

o 

I

i

Z

 

 FIG. 5.108 

           Example    5.20   .    

V

CC

C

2  

C

1  

R

C

Z

o 

V

 

C

E

R

E

R

2

h

fe

h

ie

R

1

V

i

I

i

I

o

Z

 

 FIG. 5.109 

          Voltage-divider  bias  configuration.    

  

Solution:  

 

 

    a.        Z

i

R

B

7

h

ie

= 330 k

7

1.175 k

    

    

 

⬵ h

ie

1.171 k

    

   b.         r

o

=

1

h

oe

=

1

20 

mA>V

= 50 k    

        Z

o

=

1

h

oe

7

R

C

= 50 k

7

2.7 k

 = 2.56 k⍀ ⬵ R

C

    

   c.        A

v

= -

h

fe

(R

C

7 1>h

oe

)

h

ie

= -

(120)(2.7 k

7 50 k)

1.171 k

= ⴚ262.34     

   d.        A

i

⬵ h

fe

120          

  Voltage-Divider  Configuration 

 For the voltage-divider bias configuration of  Fig.   5.109   , the resulting small-signal ac 
equivalent network will have the same appearance as  Fig.   5.107   , with  

B

   replaced by 

   R

 = R

1

7

R

2

.    

 


background image

327

APPROXIMATE HYBRID 

EQUIVALENT CIRCUIT

  

i

     

  From   Fig.    5.107     with     R

B

R,    

    

Z

i

R

1

7

R

2

7

h

ie

  

(5.147)     

  

o

     

  From   Fig.    5.107   , 

    

Z

o

⬵ R

C

  

(5.148)     

  

v

     

 

    

A

v

= -

h

fe

(R

C

7 1>h

oe

)

h

ie

  

(5.149

)     

  

i

     

 

    

A

i

=

h

fe

(R

1

R

2

)

R

1

R

2

h

ie

  

(5.150)

      

  Unbypassed  Emitter-Bias  Configuration 

 For the CE unbypassed emitter-bias configuration of  Fig.   5.110   , the small-signal ac model 
will  be  the  same  as   Fig.    5.30   ,  with 

e

   replaced by  

ie

   and     

bI

b

    by    h

fe

I

b

.     The  analysis  will 

proceed in the same manner. 

 

V

CC

R

C

Z

o 

V

 

R

E

h

fe

h

ie

R

B

V

i

I

o

I

i

Z

 

 FIG. 5.110 

          CE  unbypassed  emitter-bias  configuration.    

  

i

     

 

    

Z

b

⬵ h

fe

 R

E

  

(5.151)    

 and 

    Z

i

R

B

7

Z

b

  

(5.152)     

  Z 

    

 

    

Z

o

R

C

  

(5.153)     

  

v  

   

 

    

A

v

= -

h

fe

R

C

Z

b

⬵ -

h

fe

R

C

h

fe

R

E

   

 and 

    A

v

⬵ -

R

C

R

E

  

(5.154)     


background image

BJT AC ANALYSIS

328

 

  

i

     

 

    

A

i

= -

h

fe

R

B

R

B

Z

b

  

(5.155)    

 or 

    A

i

= -A

v

 

Z

i

R

C

  

(5.156)

      

  Emitter-Follower  Configuration 

 For the emitter-follower of  Fig.   5.38   , the small-signal ac model will match that of  Fig. 
  5.111   ,  with     

br

e

h

ie

     and     

b = h

fe

.    The resulting equations will therefore be quite similar. 

 

V

CC

h

ie

h

fe

V

o

Z

 

R

E

I

o

V

i

I

i

Z

i

R

B

 FIG. 5.111 

          Emitter-follower  configuration.    

 FIG. 5.112 

          Defining   Z 

o

    for  the  emitter-follower  configuration.    

  Z 

i

     

 

    

Z

b

⬵ h

fe

R

E

  

(5.157)

    

    

Z

i

R

B

7

Z

b

  

(5.158)     

  

o

     

  For   

o

  , the output network defined by the resulting equations will appear as shown in 

 Fig.    5.112   .  Review  the  development  of  the  equations  in   Section    5.8     and 

 

    

Z

o

R

E

7

h

ie

1

h

fe

   

 or,  because     1

h

fe

⬵ h

fe

,    

    

Z

o

⬵ R

E

7

h

ie

h

fe

  

(5.159)     


background image

329

APPROXIMATE HYBRID 

EQUIVALENT CIRCUIT

  

v

     

  For the voltage gain, the voltage-divider rule can be applied to  Fig.   5.112    as follows: 

    

V

o

=

R

E

 (V

i

)

R

E

h

ie

>(1 + h

fe

)

   

 but,  since     1

h

fe

⬵ h

fe

,    

    

A

v

=

V

o

V

i

R

E

R

E

h

ie

>h

fe

  

(5.160)     

  

i

     

 

    

A

i

=

h

fe 

R

B

R

B

Z

b

  

(5.161)

    

 or 

    A

i

= -A

v

Z

i

R

E

  

(5.162)      

  Common-Base  Configuration 

 The last configuration to be examined with the approximate hybrid equivalent circuit will be 
the common-base amplifier of  Fig.   5.113   . Substituting the approximate common-base hybrid 
equivalent model results in the network of  Fig.   5.114   , which is very similar to  Fig.   5.44   . 

  

I

c 

V

EE

h

ib 

, h

fb

I

i

V

CC

V

o

Z

 

Z

i

I

o

R

E

R

C

V

i

+

+

+

+

 FIG. 5.113 

          Common-base  configuration.    

I

i

Z

 

R

E

Z

i

+

V

i

R

C

I

o

V

o

+

I

e

h

fb

h

ib

I

e

 FIG. 5.114 

         Substituting the approximate hybrid equivalent circuit into the ac equivalent network 

of   Fig.    5.113   .    

 We have the following results from  Fig.   5.114   . 

  

    

 

    

Z

i

R

E

7

h

ib

  

(5.163)

     

  Z

 o

     

 

    

Z

o

R

C

  

(5.164)     


background image

BJT AC ANALYSIS

330

 

  

v  

   

 

    

V

o

= -I

o

R

C

= -(h

f b

I

e

)R

C

   

 with 

     I

e

=

V

i

h

ib

    

 

and 

 

   V

o

= -h

f b

V

i

h

ib

R

C

    

 so  that 

    A

v

=

V

o

V

i

= -

h

f b

R

C

h

ib

  

(5.165)     

  

A

 i  

   

 

    

A

i

=

I

o

I

i

h

f b

⬵ -1  

(5.166)    

  

EXAMPLE 5.21 

  For  the  network  of   Fig.    5.115   ,  determine: 

    a.      

i

  .  

    b.       

o

  .  

    c.       

v

  .  

    d.       

i

  .   

 

3.3 k

Ω

10 V

2.2 k

Ω

4 V

I

i

V

o

Z

 

Z

i

I

o

V

i

+

+

h

fb  

=  

− 0.99

h

ib  

=  14.3 

Ω

h

ob  

=  0.5   A/V

 μ

+

+

 FIG. 5.115 

           Example    5.21   .    

  

Solution:  

 

 

    a.        Z

i

R

E

7

h

ib

= 2.2 k

7

14.3 

 = 14.21 ⍀ ⬵ h

ib

     

   b.        r

o

=

1

h

ob

=

1

0.5 

mA>V

2 M

     Z

o

=

1

h

ob

7

R

C

⬵ R

C

3.3 k

     

   c.        A

v

= -

h

f b

 R

C

h

ib

= -

(

-0.99)(3.3 k)

14.21

229.91     

   d.        A

i

⬵ h

f b

= ⴚ1        

 The remaining configurations that were not analyzed in this section are left as an exercise 

in the problem section of this chapter. It is assumed that the analysis above clearly reveals the 
similarities in approach using the  

e

   or approximate hybrid equivalent models, thereby 

removing any real difficulty with analyzing the remaining networks of the earlier sections.    

   5.21 

 COMPLETE  HYBRID  EQUIVALENT  MODEL 

 ● 

 The analysis of  Section   5.20    was limited to the approximate hybrid equivalent circuit with 
some discussion about the output impedance. In this section, we employ the complete 
equivalent circuit to show the effect of  

r

   and define in more specific terms the effect of  

o

  . 

It is important to realize that because the hybrid equivalent model has the same appearance 
for the common-base, common-emitter, and common-collector configurations, the equa-
tions developed in this section can be applied to each configuration. It is only necessary to 




رفعت المحاضرة من قبل: Dhuha Aldabbagh
المشاهدات: لقد قام 9 أعضاء و 451 زائراً بقراءة هذه المحاضرة








تسجيل دخول

أو
عبر الحساب الاعتيادي
الرجاء كتابة البريد الالكتروني بشكل صحيح
الرجاء كتابة كلمة المرور
لست عضواً في موقع محاضراتي؟
اضغط هنا للتسجيل